You are on page 1of 141

UNDER-GRADUATE PROGRAMME

B.Sc. Mathematics
Fifth Semester
DYNAMICS
B.Sc. Mathematics Core Paper XII
Third Year - Fifth Semester Dynamics

COURSE CONTENT DEVELOPED BY


Dr. G. PALANI
Assistant Professor
Department of Mathematics
Dr. Ambedkar Government Arts College
Vysarpadi, Chennai – 600 039
KINEMATICS Page 1

CHAPTER 1
KINEMATICS-I

1 Motion in a straight line under uniform ac-


celeration
Definition 1.1 (Displacement) A particle changes its position from a point
P to a point Q, then P Q is called a displacement and it’s denoted by s, i.e., P Q =
s.
P Q

Definition 1.2 (Velocity) The rate of change of displacement is called a


velocity and it is denoted by v.
ds
i.e., v=
dt
Definition 1.3 (Acceleration) The rate of change of velocity is called an
acceleration and it is denoted by a.
d2 s
 
dv d ds
i.e., a = = = 2
dt dt dt dt
Bookwork 1.4 A particle moving along a straight line starting with velocity
u and having a constant acceleration a in its direction of motion. If v is the
velocity after time t and s is the distance described by it during that time, then
v = u + at (1.1)
1
s = ut + at2 (1.2)
2
v 2 = u2 + 2as (1.3)
Proof.
A a, t B
u v

A particle moves in a straight line from a point A to a point B with velocity


u and moves with a constant acceleration a and acquired velocity v in time t.
ds d2 s
We know that and 2 are respectively the formula for velocity and accel-
dt dt
eration in time t.
Given that Acceleration is constant
d2 s
i.e., = a (constant) (1.4)
dt2

Dr G.PALANI
KINEMATICS Page 2

Integrating (1.4) on both sides with respect to time t, we get

ds
v= = at + c
dt

Where c is a constant of integration.


Initially, When t = 0, we have v = u. Thus, we get c = u

Hence, v = u + at (1.5)

Integrating (1.5) on both sides with respect to time t, we get

1
s = = ut + at2 + c1
2

Where c1 is a constant of integration.


Initially, When t = 0, we have s = 0. Thus, we get c1 = 0

1
Hence, s = ut + at2 (1.6)
2

Squaring both sides of (1.5), we get

v 2 = (u + at)2
= u2 + a2 t2 + 2uat 
1
= u2 + 2a ut + at2
2
2 2
v = u + 2as

Note 1.5 1. Average Velocity:

Initial velocity+Final velocity


Average velocity =
2
u+v u + u + at 1
= = = u + at
2 2 2
Dr G.PALANI
KINEMATICS Page 3

2. Distance described by particles in any particular second:


Space described by nth second = Space described by n seconds
− Space described by (n − 1) seconds
   
1 2 1 2
= un + an − u(n − 1) + a(n − 1)
2 2
1  2
= u + a n − (n − 1)2

2
1
= u + a(2t − 1) (1.7)
2
Example 1 A point moves with uniform acceleration and v1 , v2 , v3 denote the
average velocities in three successive intervals of time t1 , t2 , t3 . Prove that
v1 − v2 : v2 − v3 = t1 + t2 : t2 + t3
Solution: Let a be the constant acceleration.

Figure 1

Motion Along AB:


Initial velocity= U; Final velocity= U1 ; Average velocity= v1 ;
Time taken= t1
We Know that v = u + at
⇒ U1 = U + at1 (1.8)

Initial velocity+Final velocity


Ww Know that Average velocity =
2
U + U1 U + U + at1
i.e., v1 = = (By using (1.8))
2 2
1
i, e, , v1 = U + at21 (1.9)
2
Motion Along BC:
Initial velocity= U1 ; Final velocity= U2 ; Average velocity= v2 ;
Time taken= t2
We Know that v = u + at
⇒ U2 = U1 + at2 (1.10)

Dr G.PALANI
KINEMATICS Page 4

Initial velocity+Final velocity


Average velocity =
2
U1 + U2 U + at1 + U1 + at2
i.e., v2 = = (By using (1.8), (1.10))
2 2
U + at1 + U + at1 + at2
i, e, , = (By using (1.8))
2
1
i.e., v2 = U + at1 + at2 (1.11)
2
Motion Along CD:
Initial velocity= U2 ; Final velocity= U3 ; Average velocity= v3 ;
time taken= t3

We Know that v = u + at
⇒ U3 = U2 + at3 (1.12)

Initial velocity+Final velocity


Average velocity =
2
U2 + U3 U1 + at2 + U2 + at3
i.e., v3 = = (By using (1.10), (1.12))
2 2
U + at1 + at2 + U1 + at2 + at3
i, e, , = (By using (1.8)), (1.10)
2
U + at1 + at2 + U + at1 + at2 + at3
i, e, , = (By using (1.8))
2
1
i.e., v3 = U + at1 + at2 + at3 (1.13)
2
Subtract (1.11) from (1.9), we get
1 1 1
v1 − v2 = − at1 − at2 = − a (t1 + t2 ) (1.14)
2 2 2
Similarly, Subtract (1.13) from (1.11), we get
1 1 1
v2 − v3 = − at2 − at3 = − a (t2 + t3 ) (1.15)
2 2 2
Divide (1.14) by (1.15), we get
1
v1 − v2 − a (t1 + t2 )
= 2 (1.16)
v2 − v3 1
− a (t2 + t3 )
2
t1 + t2
= (1.17)
t2 + t3
Thus, we get the required results.

Dr G.PALANI
KINEMATICS Page 5

Example 2 A body starts with velocity u and moves with uniform acceleration;
If a , b and c be the spaces described in the pth , q th and rth seconds respectively,
show that a(q − r) + b(r − p) + c(p − q) = 0
Solution: We know that a particles travelled in nth second is given by
1
Distrance travelled in nth second = u + f (2t − 1) (By using (1.7))
2
1
a = u + f (2p − 1) (1.18)
2
1
b = u + f (2q − 1) (1.19)
2
1
c = u + f (2r − 1) (1.20)
2
Using (1.18), (1.19), (1.20), we get
   
1 1
a(q − r) + b(r − p) + c(p − q) = u + f (2p − 1) (q − r) + u + f (2q − 1) (r − p)
2 2
 
1
+ u + f (2r − 1) (p − q)
2
= 0

Example 3 A train travels from station A to station B, with an acceleration


f1 during the first part of the journey and a retardation f2 during the second
part. The train starts from rest at A and comes to rest at B. If T be the
2a (f1 + f2 )
time taken to travel from A to B, show that T 2 = where a is the
f1 f2
distance between the two stations.
Solution: Let a be the distance between the two stations and T be the time
taken to travel from station A to station B.

Figure 2

Motion along AC:


Initial velocity= 0; Final velocity= u; Acceleration= f1 ;

Dr G.PALANI
KINEMATICS Page 6

Time taken= t1 , Distance=AC


v = u + at ⇒ u = 0 + f1 t1 ⇒ u = f1 t1
u
⇒ t1 = (1.21)
f1
1
s = ut + at2
2
1
⇒ AC = 0(t1 ) + f1 t21
2
1 2
= f1 t
2 1
1 u2
= f1 (By using (1.21))
2 f12
u2
AC = (1.22)
2f1
Motion along CB:
Initial velocity= u; Final velocity= 0; Acceleration= −f2 ;
Time taken= t2 , Distance=CB
v = u + at ⇒ 0 = u − f2 t2 ⇒ u = f2 t2
u
⇒ t2 = (1.23)
f2
1
s = ut + at2
2
1
⇒ CB = u(t2 ) − f2 t22
2
u 1 u2
= u − f2 2 (By using (1.21))
f2 2 f2
u2
CB = (1.24)
2f2
Total distance = AC + CB
u2 u2
a = + (By using (1.22), (1.24))
2f1 2f2
u2 1
 
1
a = + (1.25)
2 f1 f2

Also, total time = t1 + t2


u u
T = + (By using (1.21), (1.23))
f1 f2
 2
2 2 1 1
i.e., T = u + (1.26)
f1 f2

Dr G.PALANI
KINEMATICS Page 7

Divide (1.26) by (1.25), we get

T2
 
1 1
= 2 + (1.27)
a f1 f2
 
f1 + f2
i.e., T 2 = 2a (1.28)
f1 f2

Thus, we get the required results.

Example 4 The speed of a train increases at a constant rate α from 0 to v and


then remains constant for an interval and finally decreases to 0 at a constant
rate β. Ifl be thetotal distance described, prove that the time total occupied
l v 1 1
is + + .
v 2 α β
Solution: Let l be the total distance described by the train. and T be the
total time occupied by the train.

Figure 3

Motion along AB:


Initial velocity= 0; Final velocity= v; Acceleration= α;
Time taken= t1 , Distance=AB

v = u + at ⇒ v = 0 + αt1 ⇒ v = αt1
v
⇒ t1 = (1.29)
α

1
s = ut + at2
2
1
⇒ AB = 0(t1 ) + αt21
2
1 2
= αt
2 1
1 v2
= α (By using (1.29))
2 α2
v2
AB = (1.30)

Dr G.PALANI
KINEMATICS Page 8

Motion along BC:

Distance = velocity × time


BC = vt2 (1.31)

Motion along CD:


Initial velocity= v; Final velocity= 0; Acceleration= −β;
Time taken: t3 , Distance=CD

v = u + at ⇒ 0 = v − βt3 ⇒ v = βt3
v
⇒ t3 = (1.32)
β

1
s = ut + at2
2
1
⇒ CD = v(t3 ) − βt23
 2
v 1 v2
= v − β 2 (By using (1.32))
β 2 β
v2
CD = (1.33)

Total distance = AB + BC + CD
v2 v2
l = + vt2 +
2α 2β
l v v
⇒ = + t2 +
v 2α 2β
l v v
⇒ t2 = − − (1.34)
v 2α 2β

Also, total time = t1 + t2 + t3


v v
T = + t2 + (By using (1.29), (1.32))
α β
Using (1.35) in (1.34), we get
v v l v v
T = + + − −
α β v 2α 2β
 
l v 1 1
T = + +
v 2 α β
Thus, we get the required results.

Dr G.PALANI
KINEMATICS Page 9

Example 5 A particle moving in a straight line with uniform acceleration


describes distances a and b in successive intervals of t1 and t2 seconds. Find
the acceleration.

Solution: Let f be the uniform acceleration. Motion along AB:

Figure 4

Initial velocity= u; Final velocity= u1 ; Acceleration= f ;


Time taken= t1 , Distance=a

v = u + at ⇒ u1 = u + f t1 (1.35)

1
s = ut + at2
2
1
⇒ a = ut1 + f t21 (1.36)
2

Motion along BC:


Initial velocity= u1 ; Final velocity= u2 ; Acceleration= f ;
Time taken= t2 , Distance=b

v = u + at ⇒ u2 = u1 + f t2
⇒ u2 = u + f t1 + f t2 (By using (1.35)) (1.37)

1
s = ut + at2
2
1
⇒ b = u1 t2 + f t22
2
1
⇒ b = (u + f t1 )(t2 ) + f t22
2
1 2
⇒ b = ut2 + f t1 t2 + f t2 (1.38)
2
Dr G.PALANI
KINEMATICS Page 10

Multiply (1.38) by t1 and (1.36) by t2 and then subtract we get


1 1
bt1 − at2 = ut1 t2 + f t21 t2 + f t1 t22 − ut1 t2 − f t21 t2
2 2
1 2 1
= f t t2 + f t1 t22
2 1 2
1
= f t1 t2 (t1 + t2 )
2
2 (bt1 − at2 )
⇒f = (1.39)
t1 t2 (t1 + t2 )
This is the required result
Example 6 A train moves in a straight line with a uniform acceleration and
describes equal distances s in two successive intervals of durations t1 and t2 .
2s (t1 − t2 )
Show that its acceleration is .
t1 t2 (t1 + t2 )
Solution: Let f be the uniform acceleration.

Figure 5

Motion along AB:


Initial velocity= u; Final velocity= u1 ; Acceleration= f ;
Time taken= t1 , Distance=s
v = u + at ⇒ u1 = u + f t1 (1.40)

1
s = ut + at2
2
1
⇒ s = ut1 + f t21 (1.41)
2
Motion along BC:
Initial velocity= u1 ; Final velocity= u2 ; Acceleration= f ;
Time taken= t2 , Distance=s
v = u + at ⇒ u2 = u1 + f t2
⇒ u2 = u + f t1 + f t2 (By using (1.40)) (1.42)

Dr G.PALANI
KINEMATICS Page 11

1
s = ut + at2
2
1
⇒ s = u1 t2 + f t22
2
1
⇒ s = (u + f t1 )(t2 ) + f t22
2
1 2
⇒ s = ut2 + f t1 t2 + f t2 (1.43)
2
Multiply (1.43) by t1 and (1.41) by t2 and then subtract we get
1 1
st1 − st2 = ut1 t2 + f t21 t2 + f t1 t22 − ut1 t2 − f t21 t2
2 2
1 2 1 2
= f t t2 + f t1 t2
2 1 2
1
= f t1 t2 (t1 + t2 )
2
2s (t1 − t2 )
⇒f = (1.44)
t1 t2 (t1 + t2 )
This is the required result.
Example 7 A lift ascends with a constant acceleration f , then with a constant
velocity and finally stops with a constant retardation f . If the total distance
traveled is s and the total time occupied isrt, show that the time for which the
4s
lift is ascending with constant velocity is t2 − .
f
Solution: Let f be the uniform acceleration.
Motion along AB:
Initial velocity= 0; Final velocity= v; Acceleration= f ;
Time taken= t1 , Distance=a
v = u + at ⇒ v = 0 + f t1
⇒ v = f t1
v
⇒ t1 = (1.45)
f
1
s = ut + at2
2
1
⇒ a = (0)t1 + f t21
2
1 2
⇒a = ft
2 1
1 v2
⇒a = f (By using (1.45))
2 f2
v2
⇒a = (1.46)
2f

Dr G.PALANI
KINEMATICS Page 12

Figure 6

Motion along BC:

Distance = velocity × time


b = vt2 (1.47)

Motion along CD:


Initial velocity= v; Final velocity= 0; Acceleration= −f ;
Time taken= t3 , Distance=c

v = u + at ⇒ 0 = v − f t3
⇒ v = f t3
v
⇒ t3 = (1.48)
f

Dr G.PALANI
KINEMATICS Page 13

1
s = ut + at2
2
1
⇒ c = (v)t3 − f t23
2
v 1 v2
⇒ c = (v) − f 2 (By using (1.48))
f 2 f
2 2
v v
⇒c = −
f 2f
2
v
⇒c = (1.49)
2f

Total distance = a + b + c
v2 v2
s = + vt2 + (By using (1.46), (1.47),(1.49))
2f 2f
2
v
s = + vt2 (1.50)
f

Total time = t1 + t2 + t3
v v
t = + t2 + (By using (1.45), (1.48))
f f
2v
= + t2
f
f (t − t2 )
⇒ = v (1.51)
2
Substitute the value of v in (1.50), we get

f 2 (t − t2 )2 1 f (t − t2 )
s = × + t2
4 f 2
4s
= (t − t2 ) [t − t2 + 2t2 ]
f
4s
= (t − t2 ) [t + t2 ]
f
4s
= t2 − t22

f
4s
⇒ t22 = t2 −
f
r
4s
⇒ t2 = t2 −
f
This is the required result.

Dr G.PALANI
KINEMATICS Page 14

Example 8 Two cars start off to race with velocities v1 and v2 and travel in
a straight line with uniform accelerations f1 and f2 . If the race ends in a dead
2 (v1 − v2 ) (v1 f2 − v2 f1 )
heat, prove that the length of the track is .
(f1 − f2 )2
Solution: Let the length of the track be s.
The race ends in a dead heat means the cars reach the destination
at the same time.
For the first car:
Initial velocity= v1 ; Acceleration= f1 ;
Time taken= t, Distance=s
1
s = ut + at2
2
1
⇒ s = v1 t + f1 t 2 (1.52)
2
For the second car:
Initial velocity= v2 ; Acceleration= f2 ;
Time taken= t, Distance=s
1
s = ut + at2
2
1
⇒ s = v2 t + f2 t 2 (1.53)
2
Subtracting (1.53) from (1.52), we get
1
0 = (v1 − v2 ) t + (f1 − f2 ) t2
  2
1
t (f1 − f2 ) t + (v1 − v2 ) = 0
2
2 (v1 − v2 )
Thus, t = 0, − (1.54)
f1 − f2
t = 0 corresponds to the initial position, hence we reject it and hence, we have
2 (v1 − v2 )
t=− . Substitute this value of t in (1.52), we get
f1 − f2
2v1 (v1 − v2 ) 1 4 (v1 − v2 )2
s = − + ×
f1 − f2 2 (f1 − f2 )2
 
2 (v1 − v2 ) f1 (v1 − v2 )
= −v1 +
f1 − f2 f1 − f2
 
2 (v1 − v2 ) v1 f2 − v2 f1
=
f1 − f2 f1 − f2
2 (v1 − v2 ) (v1 f2 − v2 f1 )
s =
(f1 − f2 )2

Dr G.PALANI
KINEMATICS Page 15

Thus we proved the result.


Example 9 If a point moves in a straight line with uniform acceleration and
covers successive equal distances in times t1 , t2 , t3 , then show that
1 1 1 3
− + = .
t1 t2 t3 t1 + t2 + t3
Solution:
Motion along AB:
Initial velocity= v1 ; Final velocity= v2 ; ;
v1 + v2
Time taken= t1 , Distance=s Mean Velocity = .
2
Distance = velocity × time
1
s = (v1 + v2 ) t1
2
s (v1 + v2 )
⇒ = (1.55)
t1 2
Motion along BC:
Initial velocity= v2 ; Final velocity= v3 ; ;
v2 + v3
Time taken= t2 , Distance=s Mean Velocity = .
2
Distance = velocity × time
1
s = (v2 + v3 ) t2
2
s (v2 + v3 )
⇒ = (1.56)
t2 2
Motion along CD:
Initial velocity= v3 ; Final velocity= v4 ; ;
v3 + v4
Time taken= t3 , Distance=s Mean Velocity = .
2
Distance = velocity × time
1
s = (v3 + v4 ) t3
2
s (v3 + v4 )
⇒ = (1.57)
t3 2
Motion along AD:
Initial velocity= v1 ; Final velocity= v4 ; ;
v1 + v4
Time taken= t1 + t2 + t3 , Distance=3s Mean Velocity = .
2
Distance = velocity × time
1
3s = (v1 + v4 ) (t1 + t2 + t3 )
2
3s (v1 + v4 )
⇒ = (1.58)
t1 + t2 + t3 2

Dr G.PALANI
KINEMATICS Page 16

(1.55)-(1.56)+(1.57), we get
s s s 1
− + = (v1 + v4 )
t1 t2 t3 2
 
1 1 1 3s
s − + = (By using (1.58))
t1 t2 t3 t1 + t2 + t3
1 1 1 3
i.e., − + =
t1 t2 t3 t1 + t2 + t3
Thus, we proved the required result.

Exercises:

1. The two ends of a train moving with a constant acceleration pass a certain
point with velocities u and v respectively. Show that the
r velocity with
1 2
which the middle of the train passes the same point is (u + v 2 ).
2
2. A body travels a distance s in t seconds. It starts from rest and ends
at rest. In the first of the journey, it moves with a constant acceleration
a, and in the second part with a constant retardation a0 . Show that
aa0 t2 = 2s (a + a0 ).
1
3. For of the distance between two stations a train is uniformly acceler-
m
1
ated and for of the distance it is uniformly retarded. It starts from rest
n
at one station and comes to rest at the other. Prove that the greatest
1 1
velocity is 1 + + times its average velocity.
m n
4. A train goes from one station to another being uniformly accelerated
from rest in the first quarter of the distance and being brought to rest
by uniform retardation during the last quarter. The middle half of the
journey is performed at a uniform speed. Show that the average speed
of the train is two-thirds of the full speed.

Dr G.PALANI
KINEMATICS Page 17

Kinematics-II

Bookwork 1.6 (Parallelogram law of velocities)


If a point has simultaneously two velocities represented, in magnitude and di-
rection, by the two adjacent sides of a parallelogram through the point, then the
diagonal of the parallelogram drawn through the point represents in magnitude
and direction by the resultant of the two velocities.
Let ~v1 and ~v2 be the two velocities at the point O and let these be represented in
magnitude and direction by OA and OB. Complete the parallelogram OACB.
The resultant of ~v1 and ~v2 is ~v1 + ~v2 .

Figure 7

Let the angle between ~v1 and ~v2 be α. Then the magnitude of the resultant
~v1 + ~v2 is
|~v1 + ~v2 |2 = (~v1 + ~v2 ) · (~v1 + ~v2 )
p
|~v1 + ~v2 | = (~v1 + ~v2 ) · (~v1 + ~v2 )
p
= ~v · ~v + ~v2 · ~v2 + 2~v1 · ~v2
q 1 1
= v12 + v22 + 2v1 v2 cos α where |~v1 | = v1 , |~v2 | = v2 .

Dr G.PALANI
KINEMATICS Page 18

Let θ be the angle between ~v1 and the resultant ~v1 + ~v2 . Then

|~v1 × (~v1 + ~v2 ) |


tan θ =
~v1 · (~v1 + ~v2 )
|~v1 × ~v1 + ~v1 × ~v2 |
=
~v1 · ~v1 + ~v1 · ~v2
|~0 + ~v1 × ~v2 |
=
v12 + v1 v2 cos α
|v1 v2 sin αn̂|
=
v1 (v1 + v2 cos α)
v1 v2 sin α
=
v1 (v1 + v2 cos α)
v2 sin α
=
v1 + v2 cos α
 
−1 v2 sin α
⇒θ = tan
v1 + v2 cos α

Corollary 1.7 If ~v1 and ~v2 are of equal magnitude, say v, then

|~v1 + ~v2 | = v 2 + v 2 + 2v 2 cos α
p
= v 2 (1 + cos α)
p
= v 4cos2 α/2
= 2v cos α/2

Figure 8

Dr G.PALANI
KINEMATICS Page 19

Corollary 1.8 If ~v1 and ~v2 are perpendicular to each other, then choosing
~i and ~j in their directions.~v1 = v1~i, v~2 = v2~j

|~v1 + ~v2 | = |v1~i + v2~j|


q
= v12 + v22 .
v2
tan θ =
v1
v2
⇒ θ = tan−1
v1

Figure 9

Resolution of velocity into its components.


Given the velocities ~v1 and ~v2 , we have the resultant as ~v1 + ~v2 .
Conversely if ~v1 +~v2 is given, the quantities ~v1 and ~v2 are said to be compo-
nents of ~v1 + ~v2 . Since infinite number of parallelogram can be formed with a
given diagonal a given velocity can be resolved into two components in infinite
number of different ways.

Bookwork 1.9 Resolve a velocity ~v into components in two given directions.

Let ê1 , ê2 be the unit vectors in the given directions. Let them make angles
α, β with ~v . Now ~v can be expressed as a linear combination of ê1 , ê2 as

~v = aê1 + bê2 (1.59)

Dr G.PALANI
KINEMATICS Page 20

Figure 10

Multiply (1.59) vectorially by ê1 , we get

ê1 × ~v = a (ê1 × ê1 ) + b (ê1 × ê2 )


i.e., v sin αn̂ = ~0 + b sin (α + β) n̂

where n̂ is the unit vector perpendicular to both ê1 , ê2 such that ê1 , ê2 , n̂ forms
a right handed triad.
v sin α
b = (1.60)
sin (α + β)

Multiply (1.59) vectorially by ê2 , we get

ê2 × ~v = a (ê2 × ê1 ) + b (ê2 × ê2 )


 
i.e., v sin β −n ˆ = a sin (α + β) −n ˆ

v sin β
a = (1.61)
sin (α + β)
v sin β v sin α
~v = ê1 + ê2 (1.62)
sin (α + β) sin (α + β)

Bookwork 1.10 Express the velocity ~v in terms of its components in two


perpendicular directions.

Dr G.PALANI
KINEMATICS Page 21

Let ~i, ~j be the unit vectors along the


perpendicular directions. Then we have
   
~ ~ ~
~v = ~v · i i + ~v · j ~i

If ~v makes an angle α, 90o − α with ~i and


~j respectively, then

~v · ~i = v · 1 · cos α
~v · ~j = v · 1 · cos (90o − α) = v sin α
~v = v cos α~i + v sin α~j

Example 10 A particle has two velocities ~v1 and ~v2 . Its resultant velocity is
equal to ~v1 in magntitude. Show that, when the velocity is ~v1 is doubled, the
new resultant is perpendicular to ~v2 .
Solution: Given that the magnitude of resultant of two velocities ~v1 and ~v2
is ~v1 .
|~v1 + ~v2 | = |~v1 |
|~v1 + ~v2 |2 = |~v1 |2
⇒ (~v1 + ~v2 ) · (~v1 + ~v2 ) = ~v1 · ~v1
⇒ ~v1 · ~v1 + ~v1 · ~v2 + ~v2 · ~v1 + ~v2 · ~v2 = ~v1 · ~v1
⇒ 2~v1 · ~v2 + ~v1 · ~v2 = 0
⇒ (2~v1 + ~v2 ) · ~v2 = 0
Thus, the resultant of 2~v1 and ~v2 is perpendicular to ~v2 .
Example 11 A particle has two velocities of equal magnitudes inclined to each
other at an angle θ. If one of them is halved, the angle between the other and the
original resultant velocity is bisected by the new resultant. Show that θ = 120o .
# » # »
Solution: Let OA and OB be the given two velocities with equal magnitudes.
i.e., OA = OB.
Complete the parallelogram OACB. OC bisects ∠AOB, since OA = OB.
1# »
If one of the velocities is halved, i.e., OB = OB 0 .
2 # » # »
Also, given that the angle between the velocities OA and OC is bisected by
the new resultant. Let it be OC 0 . Thus, C 0 is the mid-point of CC 0 , since OC 0
is the bisector of ∠AOC.
OA AC 0
= =1
OC CC 0
⇒ OA = OC

Dr G.PALANI
KINEMATICS Page 22

Figure 11

Hence 4OCA is an equilateral triangle and ∠AOC = 60o .


Therefore, ∠AOB = 120o .

Example 12 The boat capable of moving in still water with a speed of 10 km.p.h.
crosses a river, 2 km. broad, flowing with a speed of 6 km.p.h. Find (i) the
time of crossing by the shortest route and (ii) the minimum time of crossing.

Solution:

Figure 12

(i) Let A be the position of the boat on one bank and E the directly
opposite point on the other bank. Hence, OE is the shortest route. To cross

Dr G.PALANI
KINEMATICS Page 23

by the shortest route, the resultant velocity of the boat and the stream must
be along AE.
# »
let AB = velocity of the stream = 6
# »
AC = velocity of the boat = 10
# »
Thus, the resultant velocity is OC, the diagonal of the parallelogram ABDC
and it is perpendicular to AD.
From rightangled 4ABD

AD2 + AB 2 = BD2
⇒ AD2 = BD2 − AB 2
⇒ AD2 = 1002 − 62 = 64
⇒ AD = 8

i.e., The resultant velocity of the boat is 6 km.h.. Hence the time of crossing
Distance 2 1
by the shortest route , = = = hour = 15 minutes.
speed 8 4
(ii) The time of crossing is minimum when the component of the resultant
velocity of the boat in the perpendicular of the stream is maximum.
The time taken to cross the stream is
displacement perpendicular to the stream 2
= hour = 12 minutes.
speed in the direction 10

Exercises:
1. A man who can row a boat 10 km.p.h. in still water wishes to cross a
river, 1 kmbroad, flowing at 6 km.p.h.Find

(i) the time of crossing by the shortest route and


(ii) the minimum time of crossing.

2. A boat which can steam in still water with a velocity of 48 km.p.h. is


steaming with its bow pointed due east when it is carried by a current
which flows northward with a speed of 14 km.p.h.Find the actual distance
it would travel in 12 minutes.

Dr G.PALANI
KINEMATICS Page 24

2 Relative Velocity:
Definition 2.1 (Relative Velocity)
Let two particles A and B moves along a straight line and at time t their
displacements measured from O are SA and SB respectively. The velocities of
A and B are vA and vB respectively.

Figure 13

Displacement of B relative to A = SB − SA
d
Velocity of B relative to A = (SB − SA )
dt
dSB dSA
= −
dt dt
= vB − vA

Example 13 A ship sails north-east at 15 km.p.h. and to a passenger


√ on
board, the wind appears to blow from north with a velocity of 15 2 km.p.h.
Find the true velocity of the wind.

Solution: Let ~i and ~j be the unit vectors towards east and north direc-
tions respectively.
Letw1~i + w2~j be the true velocity of the wind and the velocity of the ship is
o~ o~
15 cos 45 i + sin 45 j .

Dr G.PALANI
KINEMATICS Page 25

Figure 14

Relative velocity of wind with respect to ship = velocity of wind − velocity of ship
√     
−15 2~j = w1~i + w2~j − 15 cos 45o~i + sin 45o~j
= (w1 − 15 cos 45o )~i + (w2 − 15 sin 45o ) ~j

   
15 15
−15 2~j = w1 − √ ~i + w2 − √ ~j
2 2

Equating the like-wise components, we get


15 15 √
w1 − √ = 0; w2 − √ = −15 2
2 2
15 15 √
w1 = √ ; w2 = √ − 15 2
2 2
15 15
w1 = √ ; w2 = − √
2 2
15 15
Hence, the true velocity of the wind is √ ~i − √ ~j.
2 2 s
152 152
Direction of the wind is south-east and magnitude is √ 2 + √ 2 = 15.
2 2

Dr G.PALANI
KINEMATICS Page 26

Example 14 To a cyclist riding due west at 10 km.p.h. the wind appears to


blow from south. When he doubles his speed, it appears
√ to him to blow from
south-west. Show that the speed of the wind is 10 2 km.p.h. and it is from
south-east.

Solution: Let w1~i + w2~j be the true velocity of the wind.

Figure 15

Case 1: The velocity of cyclist is 10 km.p.h. due west and the relative
velocity of wind appears from south.

Relative velocity of wind with respect to cyclist = velocity of wind − velocity of cyclist
 
µ~j = w1~i + w2~j + 10~i
= (w1 + 10)~i + w2~j

Equating like-wise components, we get

w1 + 10 = 0; w2 = µ
w1 = −10; w2 = µ (2.1)

Case 2: The velocity of cyclist is 20 km.p.h. due west and the relative
velocity of wind appears from south-west.

Relative velocity of wind with respect to cyclist = velocity of wind − velocity of cyclist
   
λ cos 45o~i + sin 45o~j = w1~i + w2~j + 20~i +
= (w1 + 20)~i + w2~j

Dr G.PALANI
KINEMATICS Page 27

Equating like-wise components, we get

λ λ
w1 + 20 = √ ; w2 = √
2 2
λ λ
−10 + 20 = √ ; w2 = √
2 2

√ 10 2
10 2 = λ; w2 = √
2
Thus, w2 = 10 (2.2)

Therefore, the true velocity of the wind is w1 = −10~i + 10~j



√Direction of the wind is from south-east and magnitude is 102 + 102 =
10 2.

Example 15 A person traveling eastward finds the wind to blow from north.
On doubling his speed he finds it to come from north-east. Show that if he
trebles his speed,the wind appear to him to come from a direction making an
1
angle θ = tan−1 north of east.
2

Solution: Let w1~i + w2~j be the true velocity of the wind.

Figure 16

Dr G.PALANI
KINEMATICS Page 28

Case 1: The velocity of person is λ~i due eastwards and the relative velocity
of wind appears from north.
Relative velocity of wind with respect to person = velocity of wind − velocity of person
 
~ ~
−µj = w1 i + w2 j − λ~i
~

= (w1 − λ)~i + w2~j


Equating like-wise components, we get
w1 − λ = 0; w2 = −µ
w1 = λ; w2 = −µ (2.3)
Case 2: The velocity of person is 2λ~i due eastwards and the relative
velocity of wind appears from north-east.
Relative velocity of wind with respect to person = velocity of wind − velocity of person
 
1~ 1 ~  
−α √ i + √ j = w1~i + w2~j − 2λ~i
2 2
= (w1 − 2λ)~i + w2~j
Equating like-wise components, we get
−α −α
w1 − 2λ = √ ; w2 = √
2 2

2λ = α; w2 = −λ (2.4)
Case 3: The velocity of person is 3λ~i due eastwards and the relative
velocity of wind makes an angle θ north-east.
Relative velocity of wind with respect to person = velocity of wind − velocity of person
   
−γ cos θ~i + sin θ~j = w1~i + w2~j − 3λ~i
= (w1 − 3λ)~i + w2~j
Equating like-wise components, we get
w1 − 3λ = −γ cos θ; w2 = −γ sin θ
−γ sin θ w2
=
−γ cos θ w1 − 3λ
−λ −λ
tan θ = =
λ − 3λ −2λ
1
tan θ =
2  
−1 1
⇒ θ = tan
2

Dr G.PALANI
KINEMATICS Page 29

Exercises
1. To a man walking at 4 km.p.h. along a road running due west, the wind
appears to blow from south, while to a cyclist travelling in the same
direction at 8 km.p.h. it appears to come from south-west. What is the
true direction and speed of the wind?

2. To a man travelling towards north-east the wind appears to come from


north. But, when he doubles his speed, the wind appears to him to come
from a direction inclined at an angle θ = cot−1 (2) east of north. Find
the true velocity of the wind.

Dr G.PALANI
KINEMATICS Page 30

3 (Relative Angular Velocity)


Definition 3.1 (Angular velocity)
If a particle P be moving along any path in a plane. If O be a fixed point in
the plane and OA a fixed straight line throughO, the rate at which the ∠AOP
increases is called the angular velocity of P about O.


ω = = θ̇
dt

Figure 17

Definition 3.2 Relative Angular Velocity


Let A1 and A2 be two particles moving in a plane. If their velocities are v1 and
v2 making angles α1 and α2 with A1 A2 , then the components in the direction
perpendicular to A1 A2 of the velocity of A2 relative to A1 is v2 sin α2 − v1 sin α2
because the velocity components of A1 and A2 in this directions are v1 sin α2
and v2 sin α2 .

Figure 18

Dr G.PALANI
KINEMATICS Page 31

v2 sin α2 − v1 sin α1
Angular velocity of A2 relative to angular velocity A1 is =
A1 A2

Note 3.3
If v2 sin α2 = v1 sin α1 , the relative angular velocity of A2 with respect to A1
becomes zero. In this case, the line A1 A2 moves parallel to itself and the linear
velocity of A2 with respect to A1 is along A1 A2 only.
Example 16 The line joining two points A, B is of constant length a and the
velocities of A, B are in directions which makes angles α and β respectively
u sin (α − β)
with AB. Prove that the angular velocity of AB is where u is the
a cos β
velocity of A.
Solution:

Figure 19

u sin α − v sin β
Relative angular velocity of B with respect to A is = (3.1)
AB
Given that AB is of constant length a, thus we have
u cos α = v cos β
u cos α
⇒v = (3.2)
cos β
Substitute (3.2) in (3.1), we get
u cos α
u sin α − sin β
cos β
Relative angular velocity of B with respect to A =
AB
u sin α cos β − u cos α sin β
=
a cos β
u sin (α − β)
=
a cos β

Dr G.PALANI
KINEMATICS Page 32

Example 17 A and B describe concentric circles of radii a and b with uniform


speeds u, v the motion being the same way round. Prove that the angular
velocity of either with respect to the other is zero when the line joining them
au + bv
subtends at the centre an angle whose cosine is .
bu + av

Solution:

(a) (b)

Figure 20

The velocities u and v makes an angle α and β with AB respectively,


∠AOB = θ where O is the common centre.
The angular velocity of A with respect to B is zero, then

v sin β = u sin α (3.3)


In, 4OAB, ∠OAB = 90o + α, ∠OBA = 90o − β

By Projection Formula, OA = OB cos θ − AB cos (90o − α)


i.e., a = b cos θ − AB sin α (3.4)
Similarly, b = a cos θ + AB sin β (3.5)
(3.4) × u + (3.5) × v ⇒ (au + bv) = [−u sin α + v sin β] AB + (bu + av) cos θ
au + bv
i.e., cos θ = (By using (3.3)) (3.6)
bu + av

Example 18 Two planets describe circles of radii a and b round the sun as
centre, with speeds varying inversely as the square roots of the radii, show
that their relative velocity vanishes
√ when the angle θ betwen the radii to those
ab
planets is given by cos θ = √ .
a − ab + b

Dr G.PALANI
KINEMATICS Page 33

Solution:
au + bv
By the above example, we have cos θ = .
bu + av
1 1
Given that u ∝ √ ; v ∝ √
a b
k k
i.e., u= √ ; v= √
a b
using (3) in(18), we get
   
k k
a √ +b √
a
cos θ =    b
k k
b √ +a √
a b
√ √ √ 
ab a+ b
= √ √
b b+a a
√ √ √ 
ab a+ b
= 3 3
(a1/2 ) + (b1/2 )
√ √ √ 
ab a+ b
a3 + b3 = (a + b) a2 − ab + b2
 
= √ √  √  By using
a + b a − ab + b

ab
= √ .
a − ab + b

Example 19 Two points A and B move with speeds v and 2v in two concen-
tric circles, centre O and radii 2r and r respectively. If both the points move
round the circles in the same sense, and if ∠OAB = α, when the relative
motion is along AB, show that cot α = 2.

Solution: Given OA = r, OB = 2r. Since the relative motion is along


AB, it follows that the relative velocity perpendicular to AB is zero.

If ∠ABO = β, then u cos α − 2u cos β = 0


1
i.e., cos β = cos α (3.7)
2
Applying sine formula to 4OAB,

OA OB
=
sin β sin α
Therefore, sin β = 2 sin α (3.8)

Dr G.PALANI
KINEMATICS Page 34

(a) (b)

Figure 21

To eliminate β, use cos2 β + sin2 β = 1.


1
using (3.7), (3.8), cos2 α + 4 sin2 α = 1
4
Multiplying throughout by 4 and dividing by sin2 α, we get

4cosec2 α = cot2 α + 16
i.e., 4 1 + cot2 α cot2 α + 16

=
⇒ 3 cot2 α = 12
i.e., cot α = 2

Example 20 Two points A and A0 describe concentric circles of radii a and a0


with angular velocities ω and ω 0 respectively. Show that angular velocity of one
(r2 + a2 − a02 ) ω + (r2 + a02 − a2 ) ω 0
about the other is where r is the distance
2r2
between them.

Solutions:
a0 ω 0 sin α0 − aω sin α
Angular velocity of AA0 = (3.9)
r
In 4OAA0 , ∠OAA0 = 90o + α, ∠OA0 A = 90o − α

By Cosine rule, a2 = a02 + r2 − 2a0 r cos (90o − α0 )


= a02 + r2 − 2a0 r sin α0
r 2 + a0 2 − a2
⇒ a0 sin α0 = (3.10)
2r
Dr G.PALANI
KINEMATICS Page 35

Figure 22

Similarly, a0 2
= a2 + r2 − 2ar cos (90o + α)
= a2 
+ r2 + 2ar sin α
a2 + r 2 − a0 2
⇒ a sin α = − (3.11)
2r

Substitute (3.10), (3.11) in (3.9), we get

r2 + a0 2 − a2 a2 + r 2 − a0 2
   
0
ω + ω
0 2r 2r
Angular velocity of AA =
r
(r2 + a2 − a02 ) ω + (r2 + a02 − a2 ) ω 0
=
2r2

Exercises

1. Two points describe concentric circles of radii a and b with speeds varying
inversely as the radii. Show that when the relative velocity between them
is parallel to the line joining the points, the angle θ between the radii of
2ab
these points is given by cos θ = 2 .
a + b2
2. Two points describe concentric circles of radii 3cms and 4cms with speeds
varying inversely as their radii. Show that the relative velocity of either
with respect to other is parallel to the line joiningbetween
 them when
24
the angle between the radii to these point is cos−1 .
25

Dr G.PALANI
KINEMATICS Page 36

3. Two particles A, B describe concentric circles of radii a and b with an-


gular speeds ω1 ,ω2 about the common centre O. Show that when
the relative angular velocity of one particle about the other vanishes,
a2 ω1 + b2 ω
cos ∠AOB = .
ab (ω1 + ω2 )

4 Coplanar Motion
Definition 4.1 When a particle moves in a plane, its motion is said to be
coplanar motion.

Bookwork 4.2 Find the components in two fixed perpendicular directions.

Let Ox and Oy be two fixed perpendicular


directions. If P (x, y) is the position of the
particle, then the position vector of P with
respect to O is

~ = x~i + y~j
~r = OP

Now, we get the velocity and acceleration of


the particle to be

~v = ~r˙ = ẋ~i + ẏ~j


~a = ẍ~i + ÿ~j

Thus, the components of velocity are ẋ and ẏ. Similarly, the components of
acceleration are ẍ and ÿ.

Bookwork 4.3 Find the components of the acceleration of a particle in the


tangential and normal directions.

Let P Q be the directions tangential to the path of the particle in the sense
in which the arcual distance s of the particle measured from a fixed pointA
increases. Let P R be the inward-drawn normal. If T̂ and N̂ are the unit
vectors in these directions, then we know that the velocity of the particle is
~v = ṡT̂ .
Here the unit vector T̂ is not a constant vector since it varies in direction with
time t.
The acceleration of the particle is

d~v d   dT̂
~a = = ṡT̂ = s̈T̂ + ṡ
dt dt dt
Dr G.PALANI
KINEMATICS Page 37

Figure 23

Suppose T̂ makes an angle ψ with the x axis, then we have

T̂ = cos ψ~i + sin ψ~j (4.1)

But N̂ makes an angle 90o + ψ with Ox. Then

N̂ = cos (90o + ψ)~i + sin (90o + ψ) ~j = − sin ψ~i + cos ψ~j (4.2)

Now, differentiate (4.1) with respect to t, we get

dT̂ dψ~ dψ dψ
= − sin ψ i + cos ψ ~j = N̂ (By using (4.2))
dt dt dt dt
dψ ds ṡ
= N̂ = N̂ (4.3)
ds dt ρ

where ρ is the radius of curvature of the path at P .


!
ṡN̂ ṡ2
~a = s̈T̂ + ṡ = s̈T̂ + N̂
ρ ρ

Thus, the components of acceleration of the particle in tangential and normal


ṡ2
directions are s̈ and .
ρ
Bookwork 4.4 Find the components of velocity and accelerations of a particle
in the radial and transverse directions.

Let Oxy be a fixed rectangular frame and ~j and ~j be unit vectors in the x and
y directions. Choose O as the pole and Ox as the initial line and let P be
(r, θ). Then the direction of OP in the sense in which r increases is called the

Dr G.PALANI
KINEMATICS Page 38

Figure 24

radial direction and the directions perpendicular to OP in the sense in which


θ increases is called the transverse direction.
Let êr and ês be the unit vectors in these directions.

êr = cos θ~i + sin θ~j


π  π 
Also, ês = cos + θ ~i + sin + θ ~j = − sin θ~i + cos θ~j
2 2

dêr
= − sin θθ̇~i + cos θθ̇~j = θ̇ês
dt
dês
= − cos θθ̇~i − sin θθ̇~j = −θ̇êr
dt

The position vector of P with respect to O is ~r = rêr . So the velocity of P at


time t is
 
~v = ṙêr + r θ̇ ês (4.4)

Thus, the components of the velocity in the radial and transverse directions
are ṙ and rθ̇ respectively.

Dr G.PALANI
KINEMATICS Page 39

The acceleration of the particle at any time t is given by


d~v
~a =
dt
   
dêr dês
= r̈êr + ṙ + ṙθ̇ês + rθ̈ês + rθ̇
dt dt
   
= r̈êr + ṙθ̇ês + ṙθ̇ês + rθ̈ês − rθ̇θ̇êr
   
2
= r̈ − rθ̇ êr + 2ṙθ̇ + rθ̈ ês

2
 1d  2 
= r̈ − rθ̇ êr + r θ̇ ês (4.5)
r dt
The components of acceleration in the radial and transverse directions are
1d  2 
r̈ − rθ̇2 and r θ̇ respectively.
r dt

5 Exercises
1. If the angular velocity of a point moving on a plane curve is constant
about a fixed origin, show that its transverse acceleration is proportional
to its radial velocity.

2. A particle moves so that the radial and transverse components of its


velocity are ar and bθ. Show that the radial and transverse components
2 b2 θ 2 b2 θ
of its acceleration are a r − , abθ + .
r r

Dr G.PALANI
CHAPTER 2
SIMPLE HARMONIC MOTION

1 Simple Harmonic Motion in a straight line


Definition 1.1 (Simple Harmonic Motion) If a particle moves along a
straight line in such a manner that its acceleration is always directed towards
a fixed point on the line and varies as the distance from the fixed point, the
particle is said to execute a simple harmonic motion.

By defintion, ẍ ∝ x (1.1)
⇒ ẍ = −n2 x (1.2)

This is the equation of motion of the particle.

Figure 1

Bookwork 1.2 To find the solution of the equation of Simple Harmonic Mo-
tion.

Let O be the fixed point on the straight line A0 OA and P , the position of
the particle at time t so that OP = x. As the acceleration of the particle at
P is proportional to OP in the direction P O, thus we have

ẍ = −n2 x (where n2 is a constant)


D2 + n2 x = 0


This is a second order linear differential equation with constant coefficent.


Hence the solution of the equation is

x = A cos nt + B sin nt (1.3)


2

Differentiate (1.3) with respect to t, we get


ẋ = −nA sin nt + nB cos nt (1.4)
Suppose the particle starts from rest at A where OA = a
i.e., initially x = a, ẋ = 0 when t = 0. Thus, we get
(1.4) ⇒ 0 = +nB cos(0)
⇒ B = 0 (1.5)
Similarly from (1.3), we get 0 = A cos(0)
⇒ A = a (1.6)
Using (1.5) and (1.6) in (1.3), we get

x = a cos nt (1.7)

This will gives the displacement of particle at any time t. Using (1.5) and
(1.6) in (1.4), we get
ẋ = −na sin nt
ẋ2 = n2 a2 sin2 nt = n2 a2 1 − cos2 nt


= n2 a2 − a2 cos2 nt


v 2 = n 2 a2 − x 2

⇒ ( By using (1.7)) (1.8)

This will gives the velocity of particle at any time t.


Definition 1.3 (Maximum speed) As t increases from zero, x decreases
from a and the speed increases from zero. The speed is maximum when x = 0.
Hence the maximum speed is na. (By using (1.8))

Definition 1.4 (Oscillation) One complete motion of the particle from a


point on its path to one extremity of its path, then to the other extremity and
back to the point, is called an oscillation.

Definition 1.5 (Vibration) The motion of the particle from one extremity
to the other extremity of its path, is called a vibration.

Definition 1.6 (Amplitude) The maximum distance through which the par-
ticle moves on either side of the mean position of the motion, is called the
amplitude of the motion (OA = a is the amplitude). In an oscillation the
particle travels along a distance equal to 4 times amplitude.
3

Definition 1.7 (Period) The time taken by the particle to make one oscil-
lation is called the period of the motion.

Let to be the time taken by the particle to move from A to O and T be the
period of oscillation. Then T = 4 × to .

At O, x = 0, t = t0
x = a cos nt ⇒ 0 = a cos nt0
π
⇒ nto =
2
π
⇒ to =
2n

π 2π
∴ T = 4 × to = 4 × = (1.9)
2n n

Definition 1.8 (Frequency) The number of oscillations per second is called


the frequency of the motion, that is the frequency is the reciprocal of the period.
So it is

1 n
f= or f=
T 2π

Definition 1.9 (Phase and epoch) The general form of the displacement x
of the particle is

x = a cos (nt + )

Here nt +  is called the phase at time t. The initial phase, that is, the phase
when t = 0 is called epoch. So  is the epoch.

2 Composition of two simple harmonic mo-


tions of same period
Bookwork 2.1 Show that the resultant of two simple harmonic motions of
same period along the same straight line is also simple harmonic with the same
period.
4

Let the displacements in the two motions be

x1 = a1 cos (nt + 1 )
x2 = a2 cos (nt + 2 )

Then the resultant displacement is given by

x = x1 + x2 = a1 cos (nt + 1 ) + a2 cos (nt + 2 )


= a1 (cos nt cos 1 − sin nt sin 1 ) + a2 (cos nt cos 2 − sin nt sin 2 )
= (a1 cos 1 + a2 cos 2 ) cos nt − (a1 sin 1 + a2 sin 2 ) sin nt
= (a cos ) cos nt − (a sin ) sin nt
= a cos (nt + )

where

a cos  = a1 cos 1 + a2 cos 2


a sin  = a1 sin 1 + a2 sin 2
q
a = (a1 cos 1 + a2 cos 2 )2 + (a1 sin 1 + a2 sin 2 )2
q
= a21 + a22 + 2a1 a2 cos2 (1 − 2 )
 
−1 a1 sin 1 + a2 sin 2
and  = tan
a1 cos 1 + a2 cos 2

Bookwork 2.2 Show that the resultant motion of two simple harmonic mo-
tions of same period along two perpendicular lines, is along an ellipse.
Choose the line of motions be x and y axes.
The displacement of first particle is x = a cos nt and the displacement of
second particle is y = b cos (nt + ).
Eliminate t between these two equations, we get

y = b (cos nt cos  − sin nt sin )


r !
x x2
= b cos  ± 1 − 2 sin 
a a
r
y x x2
= cos  ± 1 − 2 sin 
b ar a
2
y x x
− cos  = ± 1 − 2 sin 
b a a
2
y x  
2 x
− cos  = 1 − 2 sin2 
b a a
5

y 2 x2 2 2xy 2 x2
+ cos  − cos  = sin  − sin2 
b2 a2 ab a2
x2  2xy y2
cos2  + sin2  − cos  + 2 = sin2 
a2 ab b
x2 2xy y2
− cos  + 2 = sin2 
a2 ab b

This is of the form Ax2 + 2Hxy + By 2 = 0.


1 1 cos 
Where A = 2 ; B = 2 ; H = − .
a b ab

cos2  1
H 2 − AB = −
a2 b 2 a2 b 2
1
= − 2 2 1 − cos2 

ab
1
= − 2 2 sin2  < 0
ab

Hence the resultant displacement equation represents an ellipse.

3 Examples
Example 1 A particle moves in a straight line. If v is its velocity, when at
a distance x from a fixed point in the line and if v 2 = α − βx2 , where α and
β are constants. Show that the motion is simple harmonic and determine its
period and amplitude.

Solution: Given that v 2 = α − βx2 . i.e., ẋ2 = α − βx2 .


Differentiate with respect to time t, we get

2ẋẍ = 0 − β2xẋ
2ẋẍ = −β2xẋ
⇒ ẍ = −βx

Thus, the motion is simple harmonic.

To find period and amplitude:

Amplitude of simple harmonic is given by the value of x when velocity is


6

zero. i.e., v = ẋ = 0.

v = 0
⇒ α − βx2 = 0
⇒ βx2 = α
α
⇒ x2 =
β
r
α
⇒x =
β

α 2π 2π
r
Thus the amplitude is and the period is given by T = =√ .
β n β

Example 2 The displacement x of a particle moving along a straight line is


given by x = a cos nt + b sin nt. Show that the motion is simple harmonic with
√ 2π
amplitude a2 + b2 and period .
n

Solution:
The displacement of a particle at any time t is given by

x = a cos nt + b sin nt (3.1)

Differentiate (3.1) with respect to time t twice, we get

ẋ = −na sin nt + nb cos nt (3.2)


Also, ẍ = −n2 a cos nt − n2 b sin nt
= −n2 (a cos nt + b sin nt)
⇒ ẍ = −n2 x

Thus, the motion is simple harmonic.

To find period and amplitude:


Amplitude of simple harmonic is given by the
value of x when velocity is zero. i.e., v =
ẋ = 0.

ẋ = 0
⇒ − na sin nt + nb cos nt = 0
⇒ na sin nt = nb cos nt Figure 2
b
⇒ tan nt =
a
7

From the Figure 2, we see that

b
sin nt = √ (3.3)
a2 + b 2
a
cos nt = √ (3.4)
a2 + b 2

Substitute (3.3) and (3.4) in (3.1), we get


   
a b
x = a √ +b √
a2 + b 2 a2 + b 2
a2 b2
= √ +√
a2 + b 2 a2 + b 2
2
a +b 2 √
= √ = a2 + b 2
a2 + b 2

√ 2π
Thus, the amplitude of the motion is a2 + b2 and the period is T = .
n

Example 3 A body moving with a simple harmonic motion has an amplitude


a and period T . Show that the velocity v at a distance x from the mean position
is given by v 2 T 2 = 4π 2 (a2 − x2 ) .

Solution: The velocity of a particle at any time t and period is given by


ẋ2 = v 2 = n2 a2 − x2

(3.5)

T = (3.6)
n

Eliminate n between (3.5) and (3.6), we get

 2
2 2π
a2 − x 2

v =
T
v2T 2 = 4π 2 a2 − x2


Example 4 If T be the period of the S.H.M., f its acceleration and v, velocity


at any position, show that the expression f 2 T 2 + 4π 2 v 2 is a constant.
8

Solution:
f = ẍ = −n2 x

T =
n
v = n2 a2 − x2
2

 2
2 2 2 2 4 2 2π
+ 4π 2 n2 a2 − x2

f T + 4π v = n x
n
= 4n x π + 4π 2 n2 a2 − x2
2 2 2


= 4n2 π 2 x2 + a2 − x2


= 4n2 π 2 a2 (a constant)

Example 5 Show that a particle executing S.H.M. requires one sixth of its
period to move from the position of maximum displacement to one in which
the displacement is half the amplitude.
Solution:

Figure 3

Let a be the amplitude and T be the period of the S.H.M. Let t1 be the time
taken by the particle to move from its maximum displacement (A) to its half
of the amplitude (P ). Then
x = a cos nt
a
⇒ = a cos nt1
2
1
⇒ = cos nt1
2  
−1 1 π
⇒ nt1 = cos =
2 3
π
⇒ t1 =
3n 
1 2π 1
= = T
6 n 6
9

Example 6 A particle is moving with S.H.M. and while moving from the mean
position to one extreme position its distances at three consecutive seconds are

x1 , x2 , x3 . Show that its period is  .
−1
x1 + x3
cos
2x2

Solution: Let three consecutive seconds be t − 1, t, t + 1 seconds.

x1 = a cos n (t − 1)
x2 = a cos nt
x3 = a cos n (t + 1)

x1 + x3 = a cos n (t − 1) + a cos n (t + 1)
= a [cos (nt − n) + cos (nt + n)]
= a [cos nt cos n + sin nt sin n + cos nt cos n − sin nt sin n]
= 2a cos nt cos n
= 2x2 cos n
x1 + x3
⇒ = cos n
2x2
 
x1 + x3
−1
⇒ n = cos
2x2
2π 2π
Thus, period is T = =  
n −1
x1 + x3
cos
2x2

Example 7 A particle is executing a S.H.M. of period T with O as the mean


position. The particle passes through a point P in the direction
 of OP . Show
T VT
that the time which lapses before its return to P is tan−1 .
π 2π · OP

Figure 4
10

Solution:Let the particle take a time t1 to reach the end A from P . Then
the time is taken to reach from A to P is also t1 .
Thus required time is 2t1 .
Let OP = b and OA = a. Consider the motion from A to P .

v 2 = n 2 a2 − x 2 ⇒ V 2 = n 2 a2 − b 2
 

x = a cos nt ⇒ b = a cos nt1



b a2 − b 2 V
i.e., cos nt1 = ⇒ tan nt1 = =
a b nb

2π 1 T
Now, period T = ⇒ =
n n 2π
VT
Thus, tan nt1 =
2πb

 
VT
−1
⇒ nt1 = tan
2πb
   
1 −1 V T T −1 V T
i.e., t1 = tan = tan
n 2πb 2π 2πb
 
T VT
Thus, required time = 2t1 = tan−1
π 2π · OP

Example 8 A particle moves in a S.H.M. in a straight line. In the first


second, after starting from rest, it travels a distance a and in the next second,
it travels a distance b in the same direction. Prove that the amplitude of the
2a2
motion is .
3a − b

Figure 5

Solution: Let α be the amplitude.

At t = 1, x = a cos nt ⇒ α − a = α cos n
At t = 2, x = a cos nt ⇒ α − a − b = α cos 2n
11

α − a − b = α cos 2n
⇒ α − a − b = α 2 cos2 n − 1
 
"  2 #
α−a
= α 2 −1
α
" #
2
2 (α − a)
= α −1
α2
" #
2 (α − a)2 − α2
= α
α2
⇒ α2 − bα − aα 2 (α − a)2 − α2
=
α2 − bα − aα 2 α2 − 2aα + a2 − α2

=
⇒ 4aα − bα − aα 2a2
=
⇒ (3a − b) α 2a2
=
2a2
⇒α =
3a − b

Example 9 A particle is executing a S.H.M. with O as the mean position


and a as the amplitude. When it is at a distance a/2 from O, its velocity is
quadrapled by a blow. Show that its new amplitude is 7a/2.

Figure 6

Solution: Let v and 4v be the velocities before and after the blow. Let
a1 be the new amplitude.
Before Blow:

v 2 = n 2 a2 − x 2


a2
   2
2 2 2 2 3a
⇒v = n a − =n
4 4
12

After Blow:

v 2 = n 2 a2 − x 2


a2
 
2 2 2
⇒ 16v = n a1 −
4
 2
a2
 
2 3a 2 2
⇒ 16n = n a1 −
4 4
2 2
48a a
+ = a21
4 4
49a2
a21 =
4
7a
⇒ a1 =
2

4 Exercises
1. If the distance x of a point on a straight line measured from a fixed point
on it and its velocity v are connected by the relation 4v 2 = 25 − x2 , show
that the motion is simple harmonic. Find the period and amplitude of
the motion.

2. The velocity of a particle moving in a straight


√ line, at a distance x from
a fixed point on the line, is given by v = k a2 − x2 where k and a are
constants. Show the motion is simple harmonic and find the amplitude
and periodic time.

3. The displacement x of a particle moving along a straight line is given by


x = A cos nt + B sin nt, where A, B, n are constants. Show that the
motion is simple harmonic. If A = 3, B = 4, n = 2, find its period,
amplitude, maximum velocity and maximum acceleration.

4. A particle is executing a S.H.M. with O as the mean position, as the
√ n
a 3
period and a as the amplitude. When it is at a distance from O,
2
it receives a blow
√ which increases its velocity by na. Show that the new
amplitude is a 3.
CHAPTER 3

PROJECTILES
1 Definitions
Definition 1.1 (Projectile) A particle or body projected is called a projec-
tile.
Definition 1.2 (Trajectory) The path pursued by a projectile is called the
trajectory of the projectile.
Definition 1.3 (Angle of projection) The angle of projection is the an-
gle that the direction in which the particle is initially projected makes with the
horizontal plane through the point of projection.
Definition 1.4 (Velocity of projection) The velocity of projection is
the velocity with which the particle is projected.
Definition 1.5 (Horizontal range) If O is the point of projection and If B
is the point at which the projectile hits the horizontal plane through O is called
the horizontal range.
Definition 1.6 (Range on an inclined plane) Suppose OA is a line of great-
est slope on an inclined plane, If a particle projected from O hits on OA at B
is the range on the inclined plane.
Definition 1.7 (Time of flight) The time of flight is the interval of time
that elapsed from the instant of projection till the instant where the particle
again meets the horizontal plane through the point of projection.
Bookwork 1.8 Show that the path of a projectile is a parabola.
Let a particle be projected from a point O with velocity u makes an angle θ
with the horizontal and the particle hit the horizontal plane at B through the
point of projection.
The velocity u has two components u cos θ horizontally which remains con-
stants and u sin θ vertically which is subjected to an acceleration.
x = horizontal distance travelled in time t
i.e., x = u cos θt (1.1)

y = vertical distance travelled in time t


1
i.e., y = u sin θt − gt2 (1.2)
2
2

Figure 1

Eliminate time t between (1.1) and (1.2), we get


x
(1.1) ⇒ t = (1.3)
u cos θ

Substitute (1.3) in (1.2), we get


 x  1  x 2
y = u sin θ − g
u cos θ 2 u cos θ

gx2
⇒ y = x tan θ −
2u2 cos2 θ

This is the equation of projectile.


Multiplying both sides by 2u2 cos2 θ, we get

sin θ
2u2 cos2 θy = x × 2u2 cos2 θ − gx2
cos θ
gx2 − 2u2 sin θ cos θx = −2u2 cos2 θy
2u2 sin θ cos θx 2u2 cos2 θy
⇒ x2 − = −
g g
3

u4 sin2 θ cos2 θ
Adding both sides by , we get
g2
2 2u2 sin θ cos θx u4 sin2 θ cos2 θ 2u2 cos2 θy u4 sin2 θ cos2 θ
x − + = − +
g g2 g g2
2
u2 sin θ cos θ 2u2 cos2 θ u2 sin2 θ
   
⇒ x− = − y−
g g 2g
2
⇒ X = −4aY (1.4)
2
u sin θ cos θ
where X = x −
g
2 2
2u cos θ
4a =
g
u2 sin2 θ
Y = y−
2g
The equation(1.4) represents a parabola
 with vertex upwards.
u2 sin θ cos θ u2 sin2 θ
Vertex is , .
g 2g
2u2 cos2 θ 2
Latus rectum is = (u cos θ)2 .
g g

2 Some Important results on the motion of a


projectile
In this section, we shall derive some important results like angle of projection,
time of flight horizontal range and greatest height.

Time of flight: Let T be the time taken by the particle to move


from O to B. At B, vertical distance travelled by the particle is zero. Consider
the vertical motion.
Intial velocity = u sin θ
Final velocity = 0
Time = T
Acceleration = −g
1 1
s = ut + f t2 ⇒ 0 = u sin θT − gT 2
2 2

2u sin θ
T =
g
4

Horizontal range: The distance between the point of pro-


jection and where the particle strikes the horizontal plane through the point
of projection is horizontal range (=OB = R). At B, the horizontal distance
travelled is OB.

horizontal distance = horizontal component velocity × time


u2 sin 2θ
 
2u sin θ
OB = u cos θ =
g g

u2 sin 2θ
R =
g

Greatest height:At the greatest height, the particle moves hor-


izontally. Let h be the greatest height. Consider the vertical motion.

Intial velocity
u sin θ =
Final velocity0 =
Acceleration −g =
u2 + 2f s ⇒ 0 = (u sin θ)2 − 2gh
v2 =
u2 sin2 θ
⇒ h =
2g

u2 sin2 θ
h (Greatest height) =
2g

Time taken by the particle to reach great-


est height: Let T be the time taken by the particle to reach greatest
1
height. At the greatest height,the particle moves horizontally. Consider the
vertical motion.

Intial velocity = u sin θ


Final velocity = 0
Time = T1
Acceleration = −g
v = u + f t ⇒ 0 = u sin θ − gT1
u sin θ
⇒ T1 =
g
5

u sin θ
T1 =
g

3 Examples
Example 1 If the greatest height attained by the particle is a quarter of its
range on the horizontal plane through the point of projection, find the angle of
projection.

Solution: Given that

1
Greatest height = × Horizontal range
4
u2 sin2 θ 1 u2 sin 2θ 1 u2 (2 sin θ cos θ)
⇒ = × = ×
2g 4 g 4 g
⇒ sin2 θ = sin θ cos θ
⇒ sin θ (sin θ − cos θ) = 0
Either sin θ = 0 or sin θ − cos θ = 0
But sin θ 6= 0
∴ sin θ − cos θ = 0
⇒ sin θ = cos θ
sin θ
⇒ = 1
cos θ
⇒ tan θ = 1
⇒θ = 45o

Example 2 If the time of flight of a shot 2isT seconds over a range of x


gT
metres, show that the elevation is tan−1 and determine the maximum
2x
height and the velocity of projection.

Solution:
6

2u sin θ
T =
g
2
u sin 2θ
x =
g
 2
2u sin θ
g
gT 2 g
=  2 
2x u sin 2θ
2
g

 2
2u sin θ g
= g × 2
g 2u sin 2θ
 2 2 
4u sin θ g
= g ×
g2 2u2 2 sin θ cos θ
sin θ
=
cos θ
gT 2
= tan θ
2x  2
−1 gT
⇒ θ = tan
2x

u2 sin2 θ
Greatest Height =
2g
(u sin θ)2
=
2g
 2
gT
2
=
2g
2 2
g T 1 gT 2
= × =
4 2g 8
2 2
u sin 2θ u 2 sin θ cos θ
Now, x = =
g g
(2u sin θ) u cos θ
=
g
⇒ x = T u cos θ
7

x
⇒ u cos θ =
T
√ p
velocity = u = u2 = u2 cos2 θ + u2 sin2 θ
s
 x 2  gT 2
= +
T 2
r
x2 g2T 2
= 2
+
rT 4
g 2 T 4 + 4x2
=
p 4T 2
g 2 T 4 + 4x2
⇒ u =
2T

Example 3 If T is the time of flight, R the horizontal range and α, the angle
of projection. Show that gT 2 = 2R tan α. If α = 60o , find in terms of R, the
height of the particle when it has moved through horizontal distance equal to
3R
.
4
Solution:
2u sin α
T = (3.1)
g
2
u sin 2α
R = (3.2)
g
 2
2u sin α
gT 2 = g
g
4u sin2 α
2
= g
g2
4u2 sin2 α
gT 2 = (3.3)
g
2
u sin 2α
2R tan α = 2 × tan α
g
4u2 sin α cos α sin α
= ×
g cos α
2 2
4u sin α
2R tan α = (3.4)
g

From (3.3) and (3.5), we get

gT 2 = 2R tan α
8

Equation of projectile is

gx2
y = x tan θ −
2u2 cos2 θ

 
3R
Since , h is a point lies on the trajectory and θ = 60o .
4

 2
3R
g
3R o 4
h = tan 60 − 2
4  2u2 cos2 60o
9R
√ g
3R 3 16
= −
4 1
2u2
√ 4
3R 3 9gR2
h = − (3.5)
4 8u2
2
u sin 2α
But R =
g
u sin 120o
2
=
g

gR 3
⇒ = (3.6)
u2 2

Substitute (3.6) in (3.5), we get

√ √
3R 3 9R 3
h = − ×
4√ 8√ 2 √
3R 3 9R 3 3R 3
= − =
4 16 16

Example 4 A particle is projected under gravity in a vertical plane with a


velocity u at an angle α to the horizontal. If the range on the horizontal be R
u2 R2 4h
and the greatest height attained be h, show that = h+ and tan α = .
2g 16h R
9

Solution:
u2 sin 2α
R =
g
u sin2 α
2
h =
2g
R2 u sin2 α u4 sin2 2α
2
2g
h+ = + ×
16h 2g g 2 16u sin2 α
2

u2 sin2 α u4 4 sin2 α cos2 α 2g


= + ×
2g g 2 16u2 sin2 α
u2 sin2 α u2 cos2 α
= +
2g 2g
2
u
=
2g
4h u2 sin2 α g
Now, = 4× × 2
R 2g u sin 2α
2
2 sin α
=
2 sin α cos α
= tan α

Example 5 A particle projected from a given point on the ground, just clears
a wall of height h at a distance a from the point of projection. If the particle
moves in a vertical plane perpendicular to the wall and if the horizontal range
Rh
is R, show that the elevation of projection is given by tan α = .
a (R − a)
Solution:
Equation of projectile is
gx2
y = x tan α −
2u2 cos2 α
Since (a, h) and (R, 0) is a point lies on the trajectory.
ga2
h = a tan α − 2 (3.7)
2u cos2 α
gR2
0 = R tan α − 2 (3.8)
2u cos2 α
2
gR
(3.8) ⇒ R tan α =
2u2 cos2 α
gR
⇒ tan α =
2u cos2 α
2
tan α g
⇒ = (3.9)
R 2u2 cos2 α
10

Figure 2

Substitute (3.9) in (3.7), we get

a2 tan α
h = a tan α −
R
 a
⇒ h = a tan α 1 −
 R
R−a
⇒ h = a tan α
R
Rh
⇒ tan α =
a (R − a)

Example 6 A particle is thrown over a triangle from one end of a horizontal


base and grazing the vertex falls on the other end of the base. If A, B are the
base angles, and α the angle of projection, show that tan α = tan A + tan B.

Solution:

From the previous example, we have

Rh
tan α = (3.10)
a (R − a)

From 4AOC, we have

h
tan A = (3.11)
a
11

Figure 3

Similarly, from 4ABC, we have

h
tan b = (3.12)
R−a

Adding (3.10) and (3.12), we get

h h
tan A + tan B = +
a R−a
Rh
= (3.13)
a (R − a)

From (3.10) and (3.13), we get

tan α = tan A + tan B

Example 7 Two particles are projected from the same point O with the same
velocity at angles α and β aimed at a target on the horizontal plane through
O. One falls a metres too short and the other b metres too far from the target.
If θ is the correct angle of projection so as to hit the target, show that
(a + b) sin θ = a sin 2β + b sin 2α.

Solution:
12

Figure 4

u2 sin 2α
Case 1: R−a = (3.14)
g
2
u sin 2β
Case 2: R+b = (3.15)
g
2
u sin 2θ
Case 3: R = (3.16)
g
(3.14) × b + (3.16) × a, we get
bu2 sin 2α au2 sin 2β
bR + aR = +
g g
2
u
(a + b) R = [b sin 2α + a sin 2β]
g
u2 sin 2θ u2
(a + b) = [b sin 2α + a sin 2β]
g g
⇒ (a + b) sin θ = a sin 2β + b sin 2α

Example 8 A ball is projected so as to just clear two parallel walls, the first
of height a at a distance b from the point of projection and the second of height
b at a distance a from the point of projection. Supposing the path of the ball to
lie in a plane perpendicular to the walls, find the range on the horizontal plane
and show that the angle of projection exceeds tan−1 3.

Solution:
13

Figure 5

Equation of projectile is
gx2
y = x tan α −
2u2 cos2 α
Since (b, a), (a, b) and (R, 0) is a point lies on the trajectory.
gb2
a = b tan α − (3.17)
2u2 cos2 α
ga2
b = a tan α − 2 (3.18)
2u cos2 α
gR2
0 = R tan α − 2 (3.19)
2u cos2 α
g
Eliminate tan α, − from (3.17), (3.18) and (3.19), we get
2u2 cos2 α

a b b 2 a b b2
2

b a a = 0 ⇒ b a
a2 = 0
0 R R2 0 1 R
Expanding the determinant along third row, we get
−1 a3 − b3 + R a2 − b2 = 0
 

a3 − b 3
⇒ R = 2
a − b2
(a − b) (a2 + ab + b2 )
R =
(a − b) (a + b)
a + ab + b2
2
i.e., R =
a+b
14

To find Angle of Projection: For this, a2 × (3.17) − b2 × (3.18), we get

a3 − b3 = a2 b tan α − b2 a tan α
(a − b) a2 + ab + b2 = (a − b) ab tan α


a2 + ab + b2
⇒ tan α =
ab
a2 − 2ab + b2 + 3ab
=
ab
2
(a − b) + 3ab
=
ab
(a − b)2
= +3
ab
⇒ tan α > 3
i.e., α > tan−1 (3).

Example 9 A particle is projected from a point O so as to pass through two


given points in the same vertical plane with O, at heights h and k above O
and at distances a andb from it on the same side. Show that the square of
ab (b − a)
the horizontal component of the velocity of projection must be g
2 (bh − ak)
and that
 2the direction of projection must make with the horizontal an angle
2

b h − a k
tan−1 .
ab (b − a)
Solution:

Equation of projectile is
gx2
y = x tan α −
2u2 cos2 α
Since (a, h)and(b, k) is a point lies on the trajectory.
ga2
h = a tan α − (3.20)
2u2 cos2 α
gb2
k = b tan α − 2 (3.21)
2u cos2 α
(3.20) × b2 − (3.21) × a2 , we get
ga2 b2 ga2 b2
b2 h − a2 k = b2 a tan α − − a 2
b tan α +
2u2 cos2 α 2u2 cos2 α
= ab (b − a) tan α
15

Figure 6

b 2 h − a2 k
⇒ tan α =
ab (b − a)
 2 2

−1 b h − a k
i.e., α = tan
ab (b − a)
(3.20) × b − (3.21) × a, we get
ga2 b gab2
bh − ak = ab tan α − − ab tan α +
2u2 cos2 α 2u2 cos2 α
gab
= (b − a)
2u2 cos2 α
2 (bh − ak) g
=
ab (b − a) u cos2 α
2

u2 cos2 α ab (b − a)
⇒ =
g 2 (bh − ak)
ab (b − a)
i.e., (u cos α)2 = g
2 (bh − ak)
ab (b − a)
square of the horizontal component velocity = g
2 (bh − ak)


Example 10 A particle is projected with a velocity 2 ag so that it just clears
two walls of equal height a which are at a distance 2a apart. Find the latus
rectum of the path and the time of passing between the two walls.
Solution:
16

Figure 7

Equation of projectile is
gx2
y = x tan α −
2u2 cos2 α

Since (x, a) is a point lie on the trajectory and u = 2 ag is the velocity of
projection.
gx2
a = x tan α − √ 2
2 2 ag cos2 α
gx2 sec2 α
i.e., a = x tan α −
8ag

Multiply both sides by 8a, we get

8a2 = 8ax tan α − x2 sec2 α


i.e., x2 sec2 α − 8ax tan α + 8a2 = 0

This is a quadratic equation in x having two roots say x1 and x2 .


8a tan α 8a sin α
sum of roots = x1 + x2 = 2
= × cos2 α = 8a sin α cos α
sec α cos α

8a2
Product of roots = x1 x2 = 2
= 8a2 cos2 α
sec α
17

But x2 − x1 = 2a. Hence,

(x2 − x1 )2 = (x2 + x1 )2 − 4x1 x2


(2a)2 = (8a sin α cos α)2 − 4 8a2 cos2 α


i.e., 64a2 cos2 α sin2 α − 32a2 cos2 α − 4a2 = 0

Divide through out by 4a2 , we get

16 cos2 α sin2 α − 8 cos2 α − 1 = 0


16 cos2 α 1 − cos2 α − 8 cos2 α − 1

i.e., = 0
i.e., 16 cos4 α − 8 cos2 α + 1 = 0
2
i.e., 4 cos2 α − 1 = 0
⇒ 4 cos2 α − 1 = 0
⇒ 4 cos2 α = 1
1
i.e., cos2 α =
4
1
i.e, cos α =
2
∴ α = 60o

2u2 cos2 α 2 1
Latus Rectum = = × 4ag × = 2a.
g g 4

Horizontal distance between two walls


Time passing between two parallel walls =
Horizontal velocity
r
2a 2a a
= =   =2 .
u cos α √ 1 g
2 ag
2

Example 11 A body is projected at an angle α to the horizon so as to just


clear 2 walls of equal height at a distance 2a from each other. Show that the
α
range is equal to 2a cot .
2
Solution:

Equation of projectile is

gx2
y = x tan α −
2u2 cos2 α
18

Figure 8

Since (x, a) is a point lie on the trajectory.


gx2
a = x tan α −
2u2 cos2 α

Multiply both sides by 2u2 cos2 α, we get


sin α
2au2 cos2 α = x × 2u2 cos2 α − gx2
cos α
i.e., gx2 − 2u2 cos α sin αx + 2au2 cos2 α = 0
This is a quadratic equation in x having two roots say x1 and x2 .
2u2 cos α sin α u2 sin 2α
sum of roots = x1 + x2 = = = R (range)
g g

2au2 cos2 α 2a cos2 α gR


Product of roots = x1 x2 = = ×
g g sin 2α
2
2a cos α gR
= × = aR cot α
g 2 sin α cos α

But x2 − x1 = 2a. Hence,


(x2 − x1 )2 = (x2 + x1 )2 − 4x1 x2
(2a)2 = R2 − 4aR cot α
i.e., R2 − 4aR cot α − 4a2 = 0
19

This is a quadratic equation in R having two roots.



4a cot α ± 16a2 cot2 α + 16a2
R =
2
4a cot α ± 4acosecα
=
2
= 2a [cot α + cosecα] (neglecting negative sign)
   
cos α 1 1 + cos α
= 2a + = 2a
sin α sin α sin α
α
2 cos2 α
= 2a × 2
α α = 2a cot 2
2 sin cos
2 2

Example 12 Show that the greatest height reached by a particle whose initial
velocity is V and angle of projection α is unaltered if V is increased to kV and
α is decreased by λ where cosecλ = k [cot λ − cot α].

Solution:
V 2 sin2 α
Case 1: Greatest Height = .
2g
k 2 V 2 sin2 (α − λ)
Case 2: Greatest Height = .
2g
Given that the greatest height are unaltered. Thus, we have

V 2 sin2 α k 2 V 2 sin2 (α − λ)
=
2g 2g
⇒ sin α = k sin (α − λ)
= k (sin α cos λ − cos α sin λ)

Divide both sides by sin α sin λ, we get


sin α k (sin α cos λ − cos α sin λ)
=
sin α sin λ sin α sin λ
⇒ cosecλ = k (cot λ − cot α)

Example 13 A particle reaches a point P of its path in time t and t0 is the


time it takes to travel from P to the horizontal plane through the point of
1
projection. Show that the height of P above the plane is gtt0 .
2
Solution:
20

Figure 9

Let h be the height of P above OA.


Equation of trajectory is
x = u cos αt
1
h = u sin αt − gt2
2

i.e., gt2 − u sin αt + 2h = 0


This is a quadratic equation in t having two roots say t and t0 .
2h
product of roots =
g
2h
⇒ tt0 =
g
gtt0
i.e., h =
2

Example 14 Two particles are projected in two different directions with same
speed so that they have equal horizontal ranges√R. If the greatest heights at-
tained by them are h1 and h2 , show that R = 4 h1 h2 .

Note 3.1 If two particles are projected with the same velocity and same hor-
izontal range in two different directions , then sum of two angles is 90o .
i.e., If α is the angle of projection of one particle, then the angle of projec-
tion of second particle is 90o − α.
21

Solution:

u2 sin 2α
R =
g
u sin2 α
2
h1 =
2g
u sin2 (900 − α)
2
u2 cos2 α
h2 = =
2g 2g
2 2 2 2
u sin α u cos α
h1 h2 = ×
2g 2g
2
p u sin α cos α
h1 h2 =
2g
2
p 2u sin α cos α u2 sin 2α
4 h1 h2 = = =R
g g

Example 15 A particle is projected with a velocity u at an angle α to the


horizontal. If after time t it is moving in a direction making an angle β to the
horizontal, show that gt cos β = u sin (α − β).

Solution: Equation of trajectory is

x = u cos αt
1
y = u sin αt − gt2
2

The direction of a particle at any time t is given by

dy
 

β = tan−1  dt 
 
dx
 dt 
−1 u sin α − gt
= tan
u cos α
u sin α − gt
tan β =
u cos α
sin β u sin α − gt
i.e., =
cos β u cos α
⇒ u sin β cos α = u sin α cos β − gt cos β
⇒ gt cos β = u (sin α cos β − sin β cos α) = u sin (α − β)
22

4 Exercises
1. If the range on the horizontal plane of a projectile is equal to the height
due to the velocity of projection, show that the angel of projection is
either 15o or 75o .

2. If the range on the horizontal plane of a projectile and the greatest


heightabove the point of projection
r are R and H respectively, show that
gR2
the velocity of projection is 2gH + .
8H
3. A shot projected with a velocity V at an inclination 45o reaches a point
A on the horizontal plane through the point of projection. Show that,
to hit a mark at a height h above A projecting the shot at the same
V2
elevation, the velocity of projection must be increased to p .
V 2 − gh
4. ABC is a triangle in a vertical plane with AB horizontal and A =
45o , B = 60o . A particle projected from A passes through √
C and falls
at B. Show that the angle of projection is given by tan θ = 3 + 1.

5. A particle is projected from a point P with a velocity of 32m per second


at an angle of 30o with the horizontal. If P Q be its horizontal range and
if the angles of elevation from P and Q at any instant of its flight be α
1
and β respectively, show that tan α + tan β = √ .
3
6. Two particles projected from a point with the same speed u at two
different directions, have equal horizontal ranges R. If T1 , T2 are their
1
times of flight, show that R = gT1 T2 .
2
23

5 Range on an inclined plane:

Figure 10

Let P be the point of projection and the particle strike the inclined plane at
Q. Then P Q is the range on the inclined plane. Let P Q = r. Draw QN
perpendicular to the horizontal plane through P . The coordinates of Q are
(r cos β, r sin β).
Equation of projectile is

gx2
y = x tan α −
2u2 cos2 α
Since (r cos β, r sin β) is a point on it. Thus, we have

g (r cos β)2
r sin β = r cos β tan α −
2u2 cos2 α
Multiplying both sides by 2u2 cos2 α and cancelling r throughout, we have

2u2 cos2 α sin β = 2u2 cos β sin α cos α − gr cos2 β


2u2 cos β sin α cos α − 2u2 cos2 α sin β
∴ r =
g cos2 β
2u2 cos α (sin α cos β − cos α sin β)
=
g cos2 β
24

2u2 cos α sin (α − β)


i.e., r =
g cos2 β

The initial velocity u can be resolved into two components (i)u cos (α − β)
along P Q, the inclined plane and (ii)u sin (α − β) perpendicular to the inclined
plane. The acceleration g can be resolved into two components (i)g cos β per-
pendicular to the inclined plane in the downward direction and (ii)g sin β along
the inclined plane towards P . Let T be the time taken by the particle to travel
from P to Q. After time T , the particle is on the inclined plane and so during
time T , the distance travelled perpendicular to the inclined plane is zero.
Consider the vertical motion
Initial velocity = u sin (α − β)
Final velocity = 0
Time = T
Acceleration = −g cos β
Distance = 0
1
s = ut + f t2
2
1
⇒ 0 = u sin (α − β) T − g cos βT 2
2

2u sin (α − β)
T =
g cos β

Note 5.1 Determine when the range on the inclined plane is maximum, given
the magnitude u of the velocity of projection.
The range r on the inclined plane is given by
2u2 cos α sin (α − β)
r =
g cos2 β
u2
= [sin (2α − β) − sin β]
g cos2 β
Now u and β are given. r is maximum, when sin (2α − β) is maximum.
i.e., sin (2α − β) = 1
π
⇒ 2α − β =
2

π β
i.e., α = +
4 2
25

π β
If the angle of projection is α = + , then we get the maximum range.
4 2
Thus the maximum range is
u2
maximum range r = (1 − sin β)
g cos2 β
u2 (1 − sin β)
=
g 1 − sin2 β


u2 (1 − sin β)
=
g (1 − sin β) (1 + sin β)
u2
=
g (1 + sin β)
Example 16 A particle is projected at an angle α with a velocity u and it
strikes up an inclined plane of inclination β at right angles to the plane. prove
that
(i) cot β = 2 tan (α − β)
(ii) cot β = tan α − 2 tan β
If the plane is struck horizontally, show that tan α = 2 tan β.
Solution:Time of flight is
2u sin (α − β)
T = (5.1)
g cos β
Since the particle strikes the inclined plate at right angles i.e., normal to the
plane, its velocity parallel to the inclined plate at the end of time T is zero.
Consider horizontal motion.
Initial horizontal component velocity u cos (α − β)
=
Final velocity =
0
Acceleration −g sin β
=
Time =
T
v u + f t ⇒ 0 = u cos (α − β) − g sin βT
=
u cos (α − β)
i.e., T = (5.2)
g sin β
From (5.1) and (5.2), we get
2u sin (α − β) u cos (α − β)
=
g cos β g sin β
⇒ cot β = 2 tan (α − β) · · · (i)
(tan α − tan β)
i.e., cot β = 2
1 + tan α tan β
26

Cross-multiplying, we get

cot β (1 + tan α tan β) = 2 tan α − 2 tan β


i.e., cot β + tan α = 2 tan α − 2 tan β
i.e., cot β = tan α − 2 tan β · · · (ii)

If the plane is struck horizontally, the vertical velocity of the projectile at the
end of time T is zero. Consider the vertical velocity.

Initial vertical component velocity =


u sin α
Final velocity 0=
Acceleration g=
Time T=
v u + f t ⇒ 0 = u sin α − gT
=
u sin α
i.e., T = (5.3)
g

From (5.1) and (5.3), we get

2u sin (α − β) u sin α
=
g cos β g
i.e., 2 sin (α − β) = sin α cos β
i.e., 2 (sin α cos β − cos α sin β) = sin α cos β
i.e., 2 sin α cos β − 2 cos α sin β = sin α cos β
ie., sin α cos β = 2 cos α sin β
i.e., tan α = 2 tan β

Example 17 A particle projected with velocity u strikes at right angles a plane


through the point of projection inclined at angles β to the horizon. Show that
(i) the height of the point struck above the horizontal plane through the point
2u2 sin2 β 2u
of projection is 2
 and that the time of flight is q .
g 1 + 3 sin β g 1 + 3 sin2 β
2u2 sin2 β
(ii) the range of the particle on the inclined plane is .
g 1 + 3 sin2 β

Solution:The condition that the particle strike the inclined plane at right
27

angles is cot β = 2 tan (α − β).

cos β 2 sin (α − β)
=
sin β cos (α − β)
p
sin (α − β) cos (α − β) sin2 (α − β) + cos2 (α − β) 1
i.e., = = p =p
cos β 2 sin β 2
cos2 β + 4 sin β 1 + 3 sin2 β
cos β 2 sin β
∴ sin (α − β) = p ; cos (α − β) = p
1 + 3 sin2 β 1 + 3 sin2 β

2u sin (α − β)
Time of flight =
g cos β
2u cos β 2u
= ×p = q
g cos β 2
1 + 3 sin β g 1 + 3 sin2 β


2u2 sin (α − β) cos α


Range on the inclined plane =
g cos2 β
2u2 sin (α − β)
= cos [(α − β) + β]
g cos2 β
2u2 sin (α − β)
= [cos (α − β) cos β − sin (α − β) sin β]
g cos2 β
" #
2
2u cos β 2 sin β cos β
= 2
p p cos β − p sin β
g cos β 1 + 3 sin β 2 2
1 + 3 sin β 1 + 3 sin2 β
2u2 sin β
=
g 1 + 3 sin2 β


vertical height of a point struck above the horizontal


plane through the point of projection = r sin β
2u2 sin β
=  sin β
g 1 + 3 sin2 β
2u2 sin2 β
=
g 1 + 3 sin2 β

CHAPTER 4

IMPACT
1 Definitions
Definition 1.1 (Direct Impact) Two bodies are said to impinge directy
when the directions of motion of each before impact is along the common nor-
mal at the point where they touch.

Definition 1.2 (Oblique Impact) Two bodies are said to impinge obliquely
if the directions of motion of either body or both is not along the common nor-
mal at the point where they touch.

Definition 1.3 (Line of Impact) The common normal at the point of con-
tact is called the line of impact. Thus, in the case of two spheres, the line of
impact is the line joining their centres.

2 Fundamental Laws of Impact:


The following three general principles hold good when two smooth moving
bodies make an impact.

Newton’s Experimental Law(Direct Impact):

Figure 1
2

When two bodies impinge directly, their relative velocity after impact bears a
constant ratio to their relative velocity before impact and is in the opposite
direction.

v2 − v1 = −e (u2 − u1 )

Newton’s Experimental Law(Oblique Impact):

Figure 2

When two bodies impinge obliquely, their relative velocity resolved velocity
resolved along their common normal after impact bears a constant ratio to
their relative velocity before impact resolved in the same direction and is of
opposite sign.

v2 cos θ2 − v1 cos θ1 = −e (u2 cos α2 − u1 cos α1 )

The constant ratio depends on the material of which the bodies are mode and
is independent of their masses. It is generally denoted by e, and is called the
coefficient (or modulus) of elasticity (or resitution)

Note 2.1 The quantity e, which is a positive number, is never greater than
unity. It lies between 0 and 1. i.e., 0 ≤ e ≤ 1.
When e = 0, bodies are said to be perfectly inelastic.
When e = 1, bodies are said to be perfectly elastic.
3

Principle of Conservation of Momentum:

The algebraic sum of the momenta of the impinging bodies after impact is
equal to the algebraic sum of their momenta before impact, all momenta being
measured along the common normal.
m1 v1 + m2 v2 = m1 u1 + m2 u2 (Direct Impact)
m1 v1 cos θ1 + m2 v2 cos θ2 = m1 u1 cos α1 + m2 u2 cos α2 (Oblique Impact)

Motion of two smooth bodies perpendicular to the line


of Impacts:

When two smooth bodies impinge, the only force between them at the time
of impact is the mutual reaction which acts along the common normal. There
is no force acting along the common tangent and hence there is no change
of velocity in that direction. Hence the velocity of either body resolved in a
direction perpendicular to the line of impact is not altered by impact.

Figure 3

v sin θ = u sin α

Bookwork 2.2 (Direct Impact of two smooth spheres:)


A smooth sphere of mass m1 impinge directly with velocity u1 on another
smooth sphere of mass m2 moving in the same direction with velocity u2 , if the
coefficent of resitution is e, to find their velocities after the impact.
4

Let v1 and v2 be their velocities after impact.

Figure 4

By Newton’s experimental law, we have


v2 − v1 = −e (u2 − u1 ) (2.1)
By the principle of conservation of linear momentum, we have
m1 v1 + m2 v2 = m1 u1 + m2 u2 (2.2)
(2.2)-(2.1)×m2 gives
v1 (m1 + m2 ) = m1 u1 + m2 u2 + em2 (u2 − u1 )
= m2 u2 (1 + e) + (m1 − em2 ) u1
m2 u2 (1 + e) + (m1 − em2 ) u1
∴ v1 = (2.3)
m1 + m2
(2.1)×m1 +(2.2) gives
v2 (m1 + m2 ) = em1 (u2 − u1 ) + m1 u1 + m2 u2 +
= m1 u1 (1 + e) + (m2 − em1 ) u2
m1 u1 (1 + e) + (m2 − em1 ) u2
∴ v2 = (2.4)
m1 + m2
Equations (2.3) and (2.4) give the velocities of the spheres after impact.
Note 2.3 If the two spheres are perfectly elastic and of equal mass then e = 1
and m1 = m2 . Then from (2.3) and (2.4), we get
m1 u2 × 2 + 0
v1 = = u2
2m1
m1 u1 × 2 + 0
v2 = = u1
2m1
5

i.e., If two equal perfectly spheres impinge directly, they interchange their
velocities.

Bookwork 2.4 (Loss of kinetic energy due to direct impact of two smooth spheres)

Two spheres of given masses with given velocities impinge directly. Show
that there is a loss of kinetic energy and to find the amount.

Let m1 , m2 be the masses of the spheres, u1 and u2 , v1 and v2 be their velocities


before and after impact and e be the coefficient of resitutuion.
By Newton’s experimental law, we have

v2 − v1 = −e (u2 − u1 ) (2.5)

By the principle of conservartion of momentum,

m1 v1 + m2 v2 = m1 u1 + m2 u2 (2.6)
m1 (v1 − u1 ) = m2 (u2 − v2 ) (2.7)

1 1
Total kinetic energy before impact = m1 u21 + m2 u22
2 2
1 1
Total kinetic energy after impact = m1 v1 + m2 v22
2
2 2

Change in K.E = initial K.E − final K.E


1 1 1 1
= m1 u21 + m2 u22 − m1 v12 − m2 v22
2 2 2 2
1  1
m1 u1 − v1 + m2 u2 − v22
2 2 2

=
2 2
1 1
= m1 (u1 − v1 ) (u1 + v1 ) + m2 (u2 − v2 ) (u2 + v2 )
2 2
1 1
= m1 (u1 − v1 ) (u1 + v1 ) + m1 (v1 − u1 ) (u2 + v2 ) from (??)
2 2
1
= m1 (u1 − v1 ) [u1 + v1 − (u2 + v2 )]
2
1
= m1 (u1 − v1 ) [u1 − u2 − (v2 − v1 )]
2
1
= m1 (u1 − v1 ) [u1 − u2 + e (u2 − u1 )] using (2.5)
2
1
= m1 (u1 − v1 ) (u1 − u2 ) (1 − e)
2
6

Now, from (2.6), m1 (u1 − v1 ) = m2 (v2 − u2 )


u1 − v1 v2 − u2 u1 − v1 + v2 − u2
∴ = =
m2 m1 m1 + m2
(u1 − u2 ) + (v2 − v1 )
i.e., each =
m1 + m2
(u1 − u2 ) − e (u2 − u1 )
=
m1 + m2
(u1 − u2 ) (1 + e)
=
m1 + m2
m2 (u1 − u2 ) (1 + e)
∴ u1 − v1 =
m1 + m2
1 m1 m2 (u1 − u2 ) (1 + e) (u1 − u2 ) (1 − e)
Change in K.E. =
2 m1 + m2
1 m1 m2 (u1 − u2 )2 (1 − e2 )
=
2 m1 + m2

As e < 1, the above expression is always positive and so the initial kinetic
energy is greater than final kinetic energy. Hence there will be a loss in the
kinetic energy.
When e = 1, i.e., only when the bodies are perfect elastic, then the above
expression becomes zero. Hence, in this case intial kinetic energy is equal to
final kinetic energy. Therefore, the total kinetic energy is unchanged by direct
impact.

3 Examples
Example 1 Two balls impinge directly and interchange their velocities after
impact. Prove that they are perfectly elastic and are of equal masses.

Solution:

By Newton’s experimental law, we have

v2 − v1 = −e (u2 − u1 )
⇒ u1 − u2 = −e (u2 − u1 )
i.e., e = 1
7

Figure 5

By the principle of conservartion of momentum,

m1 v1 + m2 v2 = m1 u1 + m2 u2
⇒ m1 u2 + m2 u1 = m1 u1 + m2 u2
i.e., m1 (u2 − u1 ) = m2 (u2 − u1 )
i.e., m1 = m2

Example 2 A ball of mass m impinges on another of mass 2m which is mov-


ing in the same direction as the first but with one-seventh of its velocity. If
e = 3/4, show that the first ball is reduced to rest after impact.
Solution:

Figure 6
8

By Newton’s experimental law, we have


v2 − v1 = −e (u2 − u1 )
 
3 1
⇒ v2 − v1 = − u1 − u1
4 7
9
i.e., v2 − v1 = u1 (3.1)
14
By the principle of conservartion of momentum,
m1 v1 + m2 v2 = m1 u1 + m2 u2
 
1
⇒ mv1 + 2mv2 = mu1 + 2m u2
7
 
1
i.e., v1 + 2v2 = u1 + 2 u1
7
9
i.e., v1 + 2v2 = u1 (3.2)
7
(3.1)×2−(3.2) gives
9 9
2v2 − 2v1 − v1 − 2v2 = u1 − u1
7 7
i.e., − 3v1 = 0 ⇒ v1 = 0
Thus, the first ball comes to rest after impact with the second ball.
Example 3 A ball of 2m impinges directly on a ball of mass m which is at
rest. If the velocity after impact of the latter ball is equal to that of the former
1
before impact, show that the coefficient of resitution is .
2
Solution:

By Newton’s experimental law, we have


v2 − v1 = −e (u2 − u1 )
⇒ u1 − v1 = −e (0 − u1 )
i.e., u1 − v1 = eu1 (3.3)
By the principle of conservartion of momentum,
m1 v1 + m2 v2 = m1 u1 + m2 u2
⇒ 2mv1 + mu1 = 2mu1 + m (0)
i.e., 2v1 + u1 = 2u1
i.e., 2v1 = u1
1
ie., v1 = u1 (3.4)
2
9

Figure 7

substitute (3.4) in (3.3), we get


1
u1 − u1 = eu1
2
1 1
i.e., u1 = eu1 ⇒ e = .
2 2

Example 4 A ball A impinges directly on an exactly equal and similar ball B


lying on a horizontal plane. If the coefficient of resitution is e, prove that after
impact, the velocity of B will be to that of A is as 1 + e : 1 − e.
Solution:

By Newton’s experimental law, we have


v2 − v1 = −e (u2 − u1 )
⇒ v2 − v1 = −e (0 − u1 )
i.e., v2 − v1 = eu1 (3.5)
By the principle of conservartion of momentum,
m1 v1 + m2 v2 = m1 u1 + m2 u2
⇒ mv1 + mv2 = mu1 + m (0)
i.e., v1 + v2 = u1 (3.6)
Adding (3.5) and (3.6), we get
2v2 = (1 + e) u1
1+e
i.e., v2 = u1 (3.7)
2
10

Figure 8

Substitute (3.7) in (3.5), we get


1+e
u1 − eu1 = v1
2
1−e
v1 = u1 (3.8)
2
Divide (3.7) by (3.8), we get
1+e
v2 u1
= 2
v1 1−e
u1
2
1+e
=
1−e
i.e., v 2 : v1 = 1+e:1−e

Example 5 A ball impinges directly on another ball, m times its mass, which
1
is moving with times its velocity in the same direction. If the impact reduces
n
the first ball to rest, prove that
m+n
(i) e = .
m(n − 1)
n
(ii) m must be greater than .
n−2
Solution:
11

Figure 9
u 
By Newton’s experimental law, we have
v − u = −e −u
 n u
⇒ v = e u−
 n
n−1
i.e., v = eu
n
 
n−1
v = eu (3.9)
n
By the principle of conservartion of momentum,
m1 v1 + m2 v2 = m1 u1 + m2 u2
u
⇒ k(0) + mkv = ku + mk
n
mu
i.e., mv = u +
n
m+n
i.e., mv = u (3.10)
n
Substitute (3.10) in (3.9), we get
 
n−1 m+n
m eu = u
n n
m+n
i.e., e = (3.11)
m (n − 1)
As e < 1, we get
m+n
< 1
m (n − 1)
⇒m+n < m (n − 1)
i.e., n < mn − m − m
i.e., n < n (n − 2) m
n
i.e., < m
n−2
n
i.e., m >
n−2
12

Example 6 The masses of three spheres A, B, C are 7m, 7, m, thier coefficient


of restitution is unity. Their centres are in a straight line and C lies between
A and B. Initially A and B are at rest and C is given a velocity in the line
of centres in the direction of A. Show that it strikes A twice and B once, and
that the final velocities of A, B, C are 21 : 12 : 1.
Solution:

Figure 10

Impact Between A and C:

Figure 11

Impact Between A and C:


By Newton’s experimental law, we have

v2 − v1 = −e (u2 − u1 )
⇒ v2 − v1 = −1 (0 − u1 )
i.e., v2 − v1 = u1 (3.12)
13

By the principle of conservartion of momentum,

m1 v1 + m2 v2 = m1 u1 + m2 u2
⇒ mv1 + 7mv2 = mu1 + m (0)
i.e., v1 + 7v2 = u1 (3.13)

Adding (3.12) and (3.13), we get


u1
8v2 = 2u1 ⇒ v2 =
4
Substitute the value of v2 in (3.12), we get
u1
− v1 = u1
4
u1
⇒ v1 = − u1
4
3u1
i.e., v1 = −
4
The negative value of v1 shows that after impact with A the ball C moves
towards B.
Impact Between C and B:

Figure 12

By Newton’s experimental law, we have

v2 − v1 = −e (u2 − u1 )
 
3u1
⇒ w2 − w1 = −1 −0
4
3u1
i.e., w 2 − w1 = − (3.14)
4
14

By the principle of conservartion of momentum,

m1 v1 + m2 v2 = m1 u1 + m2 u2
3u1
⇒ 7mw1 + mw2 = m + m (0)
4
3u1
i.e., 7w1 + w2 = (3.15)
4
(3.15)− (3.14) gives
3u1 3u1 6u1
8w2 = + =
4 4 4
3u1
i.e., w1 =
16
Substitute the value of w1 in (3.14), we get
3u1 3u1
w2 − = −
16 4
3u1 3u1 9u1
i.e., w2 = − =−
16 4 16
The negative value of w2 shows that the ball C moves towards the ball A after
impact with the ball B.
u1
The ball A moves with velocity and the ball C moves towards A with
4
9u1
velocity . The velocity of C is greater than the velocity of A, hence there
16
will be a second impact between A and C.
Impact Between A and C:

Figure 13
15

By Newton’s experimental law, we have


v2 − v1 = −e (u2 − u1 )
 
u1 9u1
⇒ w4 − w3 = −1 −
4 16
5u1
i.e., w 4 − w3 = (3.16)
16
By the principle of conservartion of momentum,
m1 v1 + m2 v2 = m1 u1 + m2 u2
9u1 u1
⇒ 7mw4 + mw3 = m + 7m
16 4
37u1
i.e., 7w4 + w3 = (3.17)
16
Adding (3.16) and (3.17), we get
5u1 37u1 42u1
8w4 = + =
16 16 16
21u1
i.e., w4 =
64
Substitute the value of w4 in (3.16), we get
21u1 5u1
− w3 =
64 16
21u1 5u1 u1
i.e., w3 = − =
64 16 64
Now the velocity of A is greater than C, therefore therewill be no chance for
further impact between A and C.
21u1 3u1 u1
Thus the final velocities of A, B, C after impact are , , respec-
64 16 64
tively.
Thus the ratio of final velocities of A, B, C is 21 : 12 : 1.
Example 7 A smooth sphere of mass m impinges obliquely on a smooth sphere
of mass M which is at rest. Show that if m = eM , the directions of motion
after impact are at right angles, where e is the coefficient of restitution.
Solution:

By Newton’s experimental law, we have


v2 cos θ2 − v1 cos θ1 = −e (u2 cos α2 − u1 cos α1 )
v2 − v1 cos θ = −e (0 − u1 cos α)
v2 − v1 cos θ = eu1 cos α (3.18)
16

Figure 14

By the principle of conservartion of momentum,


m1 v1 cos θ1 + m2 v2 cos θ2 = m1 u1 cos α1 + m2 u2 cos α2
mv1 cos θ + M v2 = mu1 cos α + M (0)
mv1 cos θ + M v2 = mu1 cos α (3.19)
(3.19) −(3.18) ×M gives
mv1 cos θ + M v2 − M v2 + M v1 cos θ = mu1 cos α − M eu1 cos α
(m + M ) v1 cos θ = (m − eM ) u1 cos α
= (eM − eM ) u1 cos α = 0
i.e., (m + M ) v1 cos θ = 0
π
⇒ cos θ = 0 ⇒ θ =
2

Example 8 A sphere of mass m moving on a horizontal plane with velocity


v impinges obliquely on a sphere of mass m0 at rest on the same plane. If
e = 1 and m = m0 , prove that the directions of motion after impact are at
right angles.

By Newton’s experimental law, we have


v2 cos θ2 − v1 cos θ1 = −e (u2 cos α2 − u1 cos α1 )
v2 − v1 cos θ = − (0 − u1 cos α)
v2 − v1 cos θ = u1 cos α (3.20)
17

Figure 15

By the principle of conservartion of momentum,


m1 v1 cos θ1 + m2 v2 cos θ2 = m1 u1 cos α1 + m2 u2 cos α2
mv1 cos θ + m0 v2 = mu1 cos α + m0 (0)
mv1 cos θ + m0 v2 = mu1 cos α (3.21)
(3.21) −(3.20) ×m0 gives
mv1 cos θ + m0 v2 − m0 v2 + m0 v1 cos θ = mu1 cos α − m0 u1 cos α
(m + m0 ) v1 cos θ = (m − m0 ) u1 cos α
= 0
0
i.e., (m + m ) v1 cos θ = 0
π
⇒ cos θ = 0 ⇒ θ =
2

Example 9 A smooth sphere impinges on another one at rest. After collison,


their directions of motion are at right angles. Show that if they are assumed
perfectly elastic their masses must be equal.

By Newton’s experimental law, we have


v2 cos θ2 − v1 cos θ1 = −e (u2 cos α2 − u1 cos α1 )
v2 − v1 cos θ = − (0 − u1 cos α)
π
v2 − v1 cos = u1 cos α
2
i.e., v2 = u1 cos α (3.22)
18

Figure 16

By the principle of conservartion of momentum,


m1 v1 cos θ1 + m2 v2 cos θ2 = m1 u1 cos α1 + m2 u2 cos α2
π
mv1 cos + m2 v2 = m1 u1 cos α + m2 (0)
2
mv1 (0) + m2 v2 = m1 u1 cos α
i.e., m2 v2 = m1 u1 cos α (3.23)
From (3.22) and (3.23) we have,
m2 u1 cos α = m1 u1 cos α
i.e., m1 = m2
Therefore, their masses must be equal.
Example 10 A sphere of mass m collides with a sphere of mass m1 at rest,
both spheres being smooth. After collision, their paths are at right angles. If e
is the coefficient of restitution, prove that m = em1 .

By Newton’s experimental law, we have


v2 cos θ2 − v1 cos θ1 = −e (u2 cos α2 − u1 cos α1 )
v2 − v1 cos θ = −e (0 − u1 cos α)
π
v2 − v1 cos = u1 cos α
2
i.e., v2 = eu1 cos α (3.24)
19

Figure 17

By the principle of conservartion of momentum,

mv1 cos θ1 + m1 v2 cos θ2 = mu1 cos α1 + m1 u2 cos α2


π
mv1 cos + m1 v2 = m1 u1 cos α + m2 (0)
2
mv1 (0) + m1 v2 = mu1 cos α
i.e., m1 v2 = mu1 cos α (3.25)

From (3.24) and (3.25) we have,

m1 eu1 cos α = mu1 cos α


i.e., m = em1

4 Impact of a smooth sphere on a fixed hori-


zontal plane
Bookwork 4.1 A smooth sphere or particle whose mass is m and whose co-
efficient of restitution is e, impinges obliquely on a smooth fixed plane, to find
its velocity and direction of motion after impact.

Let p be the point at which the sphere strikes the plane. The common normal
at P is the vertical line at P passing through the centre of the sphere. Let it
be P O. This is the line of impact. Let the velocity of the sphere before impact
be u at an angle α with P O and v is its velocity after impact at an angle θ
with ON as shown in the figure.
20

The velocity of the sphere resolved in a direction parallel to the plane is


unaltered by the impact.
v sin θ = u sin α (4.1)

Figure 18

By Newton’s experimental law, we have


v cos θ − 0 = −e (−u cos α − 0)
i.e., v cos θ = eu cos α (4.2)
Squaring (4.1) and (4.2) and then adding, we have
v 2 cos2 θ + sin2 θ = u2 sin2 α + e2 cos2 α
 

i.e., v 2 = u2 sin2 α + e2 cos2 α



p
v = u sin2 α + e2 cos2 α (4.3)
Dividing (4.2) by (4.1), we get
cot θ = e cot α (4.4)
Hence (4.3) and (4.4) give the velocity and direction of motion of sphere after
impact.
Corollary 4.2 If e = 1, then from (4.3) we have v = u and from (4.4) we
have θ = α.
Hence if a perfectly sphere impinges on a fixed smooth plane, its velocity
is not altered by impact and the angle of reflection is equal to the angle of
incidence.
21

Corollary 4.3 If the impact is direct, we have α = 0. Then θ = 0 and


from (4.3), v = eu. Hence if an elastic sphere strikes a plane normally with
velocity u, it will rebound in the same direction with velocity eu.

Example 11 A particle falls from a height h upon a fixed horizontal plane. If


e be the coefficient of restitution, show
 that the
 whole distance described before
1 + e2
the particle has finished rebound is h . Show also that the whole time
r 1 − e2
1 + e 2h
taken is .
1−e g

Solution:

Figure 19

Let a particle falls freely from a height h in time t. The velocity of the particle
on reaching the horizontal plane be v.

v 2 = u2 + 2f s
⇒ v 2 = 0 + 2gh
v2 (velocity)2
i.e., v 2 = 2gh ⇒ h = = (4.5)
2g 2g
v = u + f t ⇒ v = 0 + gt
v velocity
v = gt ⇒ t = = (4.6)
g g

The particle hitting the horizontal plane with velocity v and rebounds with
22

velocity ev. If h1 is the maximum height reached by the particle.

v 2 = u2 + 2f s ⇒ 0 = (ev)2 − 2gh1
e2 v 2
i.e., e2 v 2 = 2gh1 ⇒ h1 = = e2 h
2g
v = u + f t ⇒ 0 = ev − gt1
ev
i.e., t1 = = et
g

Thus the time taken to fall for the first time = t.


Time taken between the first and second rebounds = 2t1 = 2et
Similarly time taken between the second and third rebounds = 2t2 = 2e2 t and
so on.

∴ total time taken = t + 2et + 2e2 t + . . .


= t + 2et 1 + e + e2 + . . .


2et
= t + 2et (1 − e)−1 = t +
  1−e
1 − e + 2e
= t
1−e
 
1+e
= t
1−e
 
1+e v
= (By using (4.5))
1−e g
 √
1+e 2gh
=
1−e g
 s
1+e 2h
=
1−e g

Total distance described by the


particle before ceases to rebound = h + 2h1 + 2h2 + · · ·
= h + 2e2 h + 2e4 h + . . .
= h + 2e2 h 1 + e2 + e4 + . . .
 
−1
= h + 2e2 h 1 − e2
2e2 h
= h+ 2
 1−e 2 
2e
= h 1+
1 − e2
23

1 − e2 + 2e2
 
= h
1 − e2
1 + e2
 
= h
1 − e2

Example 12 A ball is thrown from a point on a smooth horizontal ground with


a speed u at an angle α to the horizon. If e be the coefficient of restitution,
2u sin α
show that the total time for which the ball rebounds on the ground is
g (1 − e)
2
u sin 2α
and the horizontal distance travelled by it is .
g (1 − e)

Solution:

Figure 20

Let a particle be projected from a point O with velocity u at an angle α to the


horizon. The particle strike the horizontal plane at A1 with velocity u sin α
and rebounds with velocity eu sin α. Again the particle strikes the horizontal
plane at A2 with velocity eu sin α and rebounds with velocity e2 u sin α and so
24

on.,
2u sin α
Total time of flight fromO to A1 =
g
2eu sin α
Total time of flight fromA1 to A2 = etc.,
g
2u sin α 2eu sin α
∴ Total time taken = + + ...
g g
2u sin α 
1 + e + e2 + . . .

=
g
2u sin α
= (1 − e)−1
g
2u sin α
=
g (1 − e)

Total horizontal distance = horizontal component velocity × total time


2u sin α
= u cos α ×
g (1 − e)
2
2u sin α cos α
=
g (1 − e)
2
u sin 2α
=
g (1 − e)

Example 13 An elastic sphere is projected from a given point O with given


velocity v at an inclination α to the horizontal and after hitting a smooth
v 2 sin 2α e
vertical wall at a distance d from O returns to O. Prove that d =
g 1+e
where e is the coefficient of restitution.
Solution:

Let an elastic sphere strike the wall at A with v cos α. The spheres starts from
O and returns to O after hitting the vertical wall at A. Let t1 be the time
taken by the sphere to travel from O to A. Then
distance = velocity × time
d = v cos α · t1 (4.7)
After hitting the sphere at A and rebounds with velocity ev cos α. Let t2 be
the time taken by the sphere to move from A to O.
distance = velocity × time
d = ev cos α · t2 (4.8)
25

Figure 21

But the vertical motion is not affected by impact and throughout the interval
t1 + t2 .
Consider the vertical motion.
Initial velocity = v sin α
Acceleration = −g
Time = t1 + t2 Distance = 0

1 1
s = ut + f t2 ⇒ 0 = v sin α (t1 + t2 ) − g (t1 + t2 )2
2 2
2v sin α
i.e., t1 + t2 =
g
d d 2v sin α
⇒ + =
v cos α ev cos α g
 
d 1 2v sin α
1+ =
v cos α e g
2
2v sin α cos α e v 2 sin 2α e
d = =
g 1+e g 1+e

Example 14 A particle falls from a height h in time t upon a fixed horizontal


plane. Prove that it rebounds and reaches a maximum height e2 h in time et.

Solution: Let a particle falls freely from a height h in time t. The velocity of
26

the particle on reaching the horizontal plane be v.

v 2 = u2 + 2f s
⇒ v 2 = 0 + 2gh
v2
i.e., v 2 = 2gh ⇒ h = (4.9)
2g
v = u + f t ⇒ v = 0 + gt
v
v = gt ⇒ t = (4.10)
g

The particle hitting the horizontal plane with velocity v and rebounds with
velocity ev. If h1 is the maximum height reached by the particle.

⇒ 0 = (ev)2 − 2gh1
v 2 = u2 + 2f s
e2 v 2
i.e., e2 v 2 = 2gh1 ⇒ h1 = = e2 h
2g
v = u + f t ⇒ 0 = ev − gt1
ev
i.e., t1 = = et
g

Example 15 A heavey ball drops from the ceiling of a room and after rebound-
ing twice from the floor reaches a height equal to one half that of the ceiling.
 1/4
1
Show that the coefficient of restitution is .
2

Solution:
Let h be the height of ceiling.
h
Therefore, height ascends after second rebound is .
2
Let v be the velocity of the particle on reaching the floor for first rebound.

v 2 = u2 + 2f s ⇒ v 2 = 0 + 2gh
p
i.e., v 2 = 2gh ⇒ v = 2gh


Thus the velocity of particle after first rebound is e 2gh. √
When the ball strikes
√ the floor a second time, its velocity is e 2gh and rebound
2
with velocity e 2gh.
27

Thus the motion for the second rebound is


 
2 2
 p
2
2 h
v = u + 2f s ⇒ 0 = e 2gh − 2g
2
 
 p
2
2 h
i.e., e 2gh = 2g
2
i.e., e4 (2gh) = gh
1
i.e., e4 =
2
 1/4
1
i.e., e =
2

5 Exercises
1. Two perfectly elastic spheres of masses m and 3m are moving with equal
momentum in the same straight line and in the same direction. Show
that the smaller sphere is reduced to rest after it strikes the other.

2. A, B, C are three smooth spheres of same size and of masses m, 2m, and
m respectively lying in a straight line on a smooth horizonal table. A
is projected along the line ABC with a velocity u. If the coefficient of
restitution is 0.5, show that after B strikes C, the velocities of A, B, C
are in the ratio 0 : 1 : 2 and that there are no further impacts.

3. There are two equal perfectly elastic balls. One is at rest and a struck
obliquely by the other. Show that, after impact, their directions of mo-
tion are at right angles.

4. A particle is projected at an elevation α with velocity v from a point in a


horizontal plane and impinges on a smooth vertical wall at a distance d
from the point of projection. If e is the coefficient of resitituion between
the wall and
 2 the particle.
 Provve that the particle rebounds to a point
v sin 2α
distant e − d from the wall.
g
5. An elastic ball of mass m falls from a height h on a fixed horizontal plane
and rebounds. Show that the loss of K.E. by the impact is mgh (1 − e2 ).
CHAPTER 5

CENTRAL ORBITS
1 Definitions
Definition 1.1 (Central force) When a particle is subject to the action of
a force which is always either towards or away from a fixed point, the particle
is said to be under the action of a central force. That is, a central force is a
force whose line of action always passes through a fixed point.
Definition 1.2 (Centre of force) A central force is a force whose line of
action always passes through a fixed point. The fixed point is called the centre
of force.
Definition 1.3 (Central Orbit) The path described by a particle under a
central force is called a central orbit.
Bookwork 1.4 Show that a central orbit is a plane curve

Figure 1

Let O be the centre of force and P be the position of the particle at time t.
~ and r̂ be the unit vector along OP , m be the mass of the particle
Let ~r = OP
andφ(r)r̂ be the central force per unit mass.

Equation of motion is
m~r¨ = mφ(r)r̂
⇒ ~r¨ = φ(r)r̂
Let us now consider
d  
~r × ~r˙ = ~r˙ × ~r˙ + ~r × ~r¨
dt
= ~0 + ~r × φ(r)r̂ = ~0 + ~0 = ~0
2

This implies that ~r × ~r˙ is a constant vector say ~c. Then ~r is always per-
pendicular to ~c. So P is always in the plane through O and perpendicular to
~c. Hence the motion of P is coplanar and the orbit is a plane curve.
Bookwork 1.5 Derive the differential equation of a central orbit in polar co-
ordinates.

Figure 2

Let F be the acceleration towards the fixed point O of a moving particle P at


the end of time t. Let the polar coordinates of P be (r, θ) with respect to O
as pole and OX as the initial line.
Thus, the equations of motion along the radial direction:
 
m r̈ − rθ̇2 = −mF
i.e., r̈ − rθ̇2 = −F (1.1)
Equation of motion along the transverse direction:
1d  2 
m· r θ̇ = 0
r dt
1d  2 
i.e., r θ̇ = 0 (1.2)
r dt
(1.4) implies that r2 θ̇ = constant. i.e, r2 θ̇ = h (constant).

h 1
∴ θ̇ = = hu2 put u =
r2 r
1 1
u = ⇒r=
r u
1
r =
u
1 du 1 du dθ 1 du du
ṙ = − 2 =− 2 = − 2 hu2 = −h
u dt  u dθ dt  u dθ dθ
2
d du d du dθ du d2 u
r̈ = −h = −h = −h 2 θ̇ = −h2 u2 2
dt dθ dθ dθ dt dθ dθ
3

Substitute the value of ṙ and r̈ in equation (1.3), we get


d2 u 1 2
−h2 u2 2
− hu2 = −F
dθ u
2
du 1
−h2 u2 2 − h2 u4 = −F
dθ u 
d2 u
−h2 u2 +u = −F
dθ2
d2 u F
u+ 2 =
dθ h2 u2

d2 u F
u+ 2
= 2 2
dθ hu

This is the differential equation of central orbit in polar coordinates.


Bookwork 1.6 Derive pedal equation (or) p-r equation of a central orbit.

Figure 3

Let F be the acceleration towards the centre of force O of the particle P


describing the central orbit. Draw OL perpendicular to the tangent at P to
the curve. Let OL = p.
From the differential calculus, we know that
 2
1 1 1 dr
= 2+ 4
p2 r r dθ
1
Putting r = , then the above equation becomes
u
 2
1 2 4 1 du
= u +u − 2
p2 u dθ
 2
du
= u2 +

4

 2
1 du
= u2 +
p2 dθ

This is the pedal equation (or) p-r equation of central orbit.


Definition 1.7 (Apse:) An apse is a point on the central orbit at which the
normal to the curve passes through the centre of force. i.e., the tangent to the
curve at the point is perpendicular to the radius vectgor through the point. So
p = r.
du
So at an apse, = 0.

Thus, the line segment joining the apse and the centre of force is called the
apsidal distance.
Type I: Given the orbit, find the law of force.
Example 1 A particle describe the orbit r = aeθ cot α under a central force,
the pole being the centre. Find the law of force.
Solution:
Given that r = aeθ cot α .
1
Put r = , we get
u
1
= aeθ cot α
u
Differentiating both sides with respect to θ, we get
1 du 1
− 2
= a cot αeθ cot α = cot α
u dθ u
du
∴ = −u cot α

Again differentiating with respect to θ, we get
d2 u du
= − cot α = u cot2 α
dθ2 dθ
d2 u
u+ 2 = u + u cot2 α

F
= u 1 + cot2 α

h u2
2

F = h2 u3 cosec2 α
h2 cosec2 α
=
r3
1
i.e., F ∝ 3
r
5

Example 2 Show that the force towards the pole under which a particle de-
scribes the curve rn = an cos nθ varies inversely as the (2n + 3)th power of the
distance fof the particle from the pole.

Solution:
Given that rn = an cos nθ .
1
Put r = , we get
u
1
n
= an cos nθ
u
u−n = an cos nθ

Taking logarithm and then differeniating with respect to θ, we get

n du 1
− = − (n sin nθ)
u dθ cos nθ
du
i.e., = u tan nθ

Again differentiating with respect to θ, we get

d2 u du
(tan nθ) + u n sec2 nθ

=
dθ dθ
= u tan2 nθ + nu sec2 nθ
d2 u
u+ = u + u tan2 nθ + nu sec2 nθ

F 2
+ nu sec2 nθ

= u 1 + tan nθ
h2 u2
= u sec2 nθ + nu sec2 nθ
= u (1 + n) sec2 nθ
a2n h2 (1 + n) a2n
i.e., F = h2 u3 (1 + n) 2n =
r r2n+3
1
i.e., F ∝ 2n+3
r

Example 3 A particle describe the orbit rn = A cos nθ − B sin nθ under a


central force, the pole being the centre. Find the law of force.

Solution:
Given that rn = A cos nθ − B sin nθ .
6

Put A = an cos α, B = an sin α


B
∴, A2 + B 2 = a2n ; tan α = .
A
rn = an cos α − an sin α sin nθ
= an cos (nθ + α)
1
Put r = , we get
u
1
n
= an cos (nθ + α)
u
u−n = an cos (nθ + α)

Taking logarithm on both sides, we get

−n log u = n log a + log cos (nθ + α)

Differentiate both sides with respect to θ, we get


1 du 1
−n = −n sin (nθ + α)
u dθ cos (nθ + α)
du
i.e., = u tan (nθ + α)

Again differentiating with respect to θ, we get

d2 u du
(tan (nθ + α)) + u n sec2 (nθ + α)

=
dθ dθ
= u tan2 (nθ + α) + nu sec2 (nθ + α)
d2 u
u+ = u + u tan2 (nθ + α) + nu sec2 (nθ + α)

F
= u 1 + tan2 (nθ + α) + nu sec2 (nθ + α)

2
hu 2

= u sec2 (nθ + α) + nu sec2 (nθ + α)


= u (1 + n) sec2 (nθ + α)
a2n h2 (1 + n) a2n
i.e., F = h2 u3 (1 + n) 2n =
r r2n+3
1
i.e., F ∝ 2n+3
r

l
Example 4 A particle describe the orbit = 1 + e cos θ under a central force,
r
the pole being the centre. Find the law of force.
7

Solution:
l
Given that = 1 + e cos θ .
r
1
Put r = , we get
u
lu = 1 + e cos θ

Differentiate both sides with respect to θ, we get


du
l = −e sin θ

du e
i.e., = − sin θ
dθ l
Again differentiating with respect to θ, we get
d2 u e
2
= − cos θ
dθ l
2
du e
u + 2 = u − cos θ
dθ l
d2 u 1 + e cos θ e
u+ 2 = − cos θ
dθ l l
1 + e cos θ − e cos θ 1
= =
l l
F 1
i.e., =
h2 u2 l
h2 u2 h2
i.e., F = = 2
l lr
1
i.e., F ∝ 2
r
Type II: Given the law of force, find the orbit.
Example 5 The law of force is µu5 and a particle is projected from √ an apse
µ
at a distance a. Find the orbit when the velocity of projection is √ .
2a2
Solution:
Given that F = µu5
d2 u F µu5 µu3
∴ u+ 2 = 2 2 = 2 2 = 2
dθ hu hu h
du
Multiplying both sides by 2 , we get

du du d2 u 2µu3 du
2u +2 =
dθ dθ dθ2 h2 dθ
8

Integrating on both sides with respect to θ, we get

 2 Z
2 du 2µ
u + = 2
u3 du
dθ h
2µ u4
= +A
h2 4 √ √
µ µ
Initially h = pv = po vo = a √ =√
2a2 2a
µ
⇒ h2 =
2a2
µ
i.e., 2a2 =
h2
1 1 du
Also, u = = and =0
r a dθ
1  1
∴ +0 = 2 2a2 +A
a2 4a4
1 1
i.e., = 4a2 · 4 + A
a2 4a
1 1
i.e., = +A
a2 a2
⇒ A = 0
 2
du  u4
∴ u2 + = 2 2a2 = a2 u4
dθ 4
 2
du
a2 u4 − u2 = u2 a2 u2 − 1

i.e., =

du √
i.e., = u a2 u2 − 1

1 du 1 dr
put u = ; then =− 2
r dθ r dθ
r √
1 dr 1 a2 a2 − r 2
− 2 = − 1 =
r dθ r r2 r2
dr
i.e., −√ = dθ
a2 − r 2

Integrating on both sides with respect to θ, we get

r
cos−1 = θ+B
a
9

If θ is measured from an apse r = a and θ = 0, we get

a
cos−1 = 0+B =B
a
−1
i.e., cos (1) = B
i.e., B = 0
r
∴ cos−1 = θ
a
r
i.e., = cos θ
a
i.e, r = a cos θ

This is the required equation of orbit.

Example 6 If the law of acceleration is 5µu3 + 8µc 2 5


√ u and the particle is pro-
3 µ
jected from an apse at a distance c with velocity , prove that the equation
c

of the orbit is r = c cos .
3

Solution:
Given that F = 5µu3 + 8µc2 u5

d2 u F 5µu3 + 8µc2 u5 µ 
= 2 5u + 8c2 u3

∴ u+ 2
= 2 2
= 2 2
dθ hu hu h

du
Multiplying both sides by 2 , we get

du du d2 u 2µ  2 3 du

2u +2 = 5u + 8c u
dθ dθ dθ2 h2 dθ

Integrating on both sides with respect to θ, we get

 2 Z
2 du 2µ 
5u + 8c2 u3 du

u + = 2
dθ h
2µ 5u2 8c2 u4
 
= + +A
h2 2 4
µ
= 2 5u2 + 4c2 u4 + A

(1.3)
h
10


3 µ √
Initially h = pv = po vo = c =3 µ
c
⇒ h2 = 9µ
µ 1
i.e., =
h2 9
1 1 du
Also, u = = and =0
r c dθ

1 1 5 2 1
∴ +0 = + 4c 4 + A
c2 9 c2 c
 
1 1 5 4
i.e., = + 2 +A
c2 9 c 2 c
1 1
i.e., = +A ⇒ A=0
c2 c2
 2
du 1
u2 + 5u2 + 4c2 u4

∴ =
dθ 9
 2
du 1 5u2 + 4c2 u4 − 9u2
5u2 + 4c2 u4 − u2 =

i.e., =
dθ 9 9
2 4 2 2
4c u − 4u 4u
c2 u2 − 1

= =
9 9
du 2u √ 2 2
i.e., = c u −1
dθ 3

1 du 1 dr
put u = ; then =− 2
r dθ r dθ
r √
1 dr 2 c2 2 c2 − r 2
− 2 = −1=
r dθ 3r r2 3r2
dr 2
i.e., − √ = dθ
c2 − r 2 3
Integrating on both sides with respect to θ, we get
r 2
cos−1 = θ+B
c 3
If θ is measured from an apse r = a and θ = 0, we get
c
cos−1 = 0+B =B
c
i.e., cos−1 (1) = B
i.e., B = 0
r 2θ
∴ cos−1 =
c 3
r 2θ 2θ
i.e., = cos ⇒ r = c cos
c 3 3
11

This is the required equation of orbit.


Example 7 A particle moves with an acceleration µ [3au4√− 2 (a2 − b2 ) u5 ] is
µ
projected from an apse at a distance (a + b) with velocity . Show that the
a+b
equation of the orbit is r = a + b cos θ.
Solution:
Given that F = µ [3au4 − 2 (a2 − b2 ) u5 ]
d2 u F µ [3au4 − 2 (a2 − b2 ) u5 ] µ 
= 2 3au2 − 2 a2 − b2 u3
 
∴ u+ 2
= 2 2
= 2 2
dθ hu hu h
du
Multiplying both sides by 2 , we get

2
du du d u 2µ  2 2 2
 3  du
2u + 2 = 3au − 2 a − b u
dθ dθ dθ2 h2 dθ
Integrating on both sides with respect to θ, we get
 2 Z
2 du 2µ  2 2 2
 3
u + = 3au − 2 a − b u du
dθ h2
u3  4
 
2µ 2 2 u
= 3a − 2 a − b +A
h2 3 4
µ 
= 2 2au3 − a2 − b2 u4 + A
 
(1.4)
h


µ √
Initially h = pv = po vo = (a + b) = µ
a+b
2
⇒ h = µ
µ
i.e., = 1
h2
1 1 du
Also, u = = and =0
r a+b dθ
1 2a (a2 − b2 )
∴ + 0 = − +A
(a + b)2 (a + b)3 (a + b)4
1 2a (a − b) (a + b)
i.e., 2 = 3 − +A
(a + b) (a + b) (a + b)4
1 2a a−b
i.e., 2 = 3 − +A
(a + b) (a + b) (a + b)3
2a − a + b
= +A
(a + b)3
1 1
i.e., 2 = +A ⇒ A=0
(a + b) (a + b)2
12

 2
2 du
2au3 − a2 − b2 u4
  
∴ u + =

 2
du
2au3 − a2 − b2 u4 − u2 = u2 2au − a2 − b2 u2 − 1
     
i.e., =

du p
i.e., = u [2au − (a2 − b2 ) u2 − 1]

1 du 1 dr
put u = ; then =− 2
r dθ r dθ
r p
1 dr 1 2a (a2 − b2 ) 2ar − (a2 − b2 ) − r2
− 2 = − − 1 =
r dθ r
√ r r2 p r2
2ar − a2 + b2 − r2 b − (r + a2 − 2ar)
2 2
= =
q r2 r2
b2 − (r − a)2
=
r2
dr
i.e., −q = dθ
2 2
b − (r − a)

Integrating on both sides with respect to θ, we get


r−a
cos−1 = θ+B
b
If θ is measured from an apse r = a + b and θ = 0, we get

a+b−a
cos−1 = 0+B =B
b
−1
i.e., cos (1) = B
i.e., B = 0
r−a
∴ cos−1 = θ
b
r−a
i.e., = cos θ ⇒ r = a + b cos θ
b
This is the required equation of orbit.

Example 8 A particle moves an acceleration


r µ (r5 − c4 r) being projected from
2µ 3
an apse at a distance c with a velocity c . Show that the path of the curve
3
is x4 + y 4 = c4 .

Solution:
13

c4 µ (1 − c4 u4 )
 
5 4 1
Given that F = µ (r − c r) = µ − =
u5 u u5
d2 u F µ (1 − c4 u4 )
∴ u+ 2 = 2 2 =
dθ hu h2 u7
du
Multiplying both sides by 2 , we get

2
du du d u 2µ (1 − c4 u4 ) 2µ  du
2u + 2 2
= 2 7
= 2 u−7 − c4 u−3
dθ dθ dθ h u h dθ
Integrating on both sides with respect to θ, we get
 2 Z
2 du 2µ −7 4 −3

u + = u − c u du
dθ h2
2µ u−6 −2
 
4u
= −c +A
h2 −6 −2
 
µ 1 4 1
= 2 − 6 +c 2 +A (1.5)
h 3u u
r r
2µ 3 2µ 4
Initially h = pv = po vo = c c = c
3 3
2µ 8
⇒ h2 = c
3
µ 3
i.e., =
h2 2c8
1 1 du
Also, u = = and =0
r c dθ 
1 3 c6
∴ +0 = − + c4 · c2 + A
c2 2c 8 3
 6
1 3 2c
i.e., = +A
c2 2c8 3
1 1
i.e., = 2 +A ⇒ A=0
c2 c
 2    
2 du µ 1 4 1 3 1 4 1
∴ u + = − 6 +c 2 = 8 − 6 +c 2
dθ h2 3u u 2c 3u u
 2
3 −1 + 3c4 u4
 
du
i.e., = − u2
dθ 2c8 3u6
−1 + 3c4 u4 − 2c8 u8 (1 − c4 u4 ) (2c4 u4 − 1)
= =
r 2c8 u6 2c8 u6
du (1 − c4 u4 ) (2c4 u4 − 1)
i.e., =
dθ 2c8 u6
14


2c4 u3 du
p = dθ (1.6)
(1 − c4 u4 ) (2c4 u4 − 1)

put 1 − c4 u4 = t2
−4c4 u3 du = 2tdt
t
i.e., c4 u3 du = − dt
2
Also c4 u4 = 1 − t2
i.e., 2c4 u4 = 2 − 2t2
i.e., 2c4 u4 − 1 = 2 − 2t2 − 1 = 1 − 2t2

Thus the (1.6) can be written as


√ t
2 dt
−p 2 = dθ
(1 − 2t2 ) (t2 )
tdt
−√ √ = dθ
2t 1 − 2t2
dt
i.e., − = dθ
√ √
r
1
2· 2 − t2
2
dt
i.e., − s 2 = 2dθ
1
√ − t2
2

Integrating on both sides with respect to θ, we get


Z
dt
− s 2 = 2dθ
1
√ − t2
2

i.e., − sin−1 2t = 2θ + B
p
i.e., − sin−1 2 (1 − c4 u4 ) = 2θ + B

If θ is measured from an apse, r = c and θ = 0, we get


s  
−1 4
1
− sin 2 1−c 4 = 0+B
c
i.e., − sin−1 (0) = 0 + B ⇒ B = 0
15

p
− sin−12 (1 − c4 u4 ) = 2θ
p
i.e., sin−1 2 (1 − c4 u4 ) = −2θ
p
i.e., 2 (1 − c4 u4 ) = sin (−2θ) = − sin 2θ

Squaring both sides, we get

2 1 − c4 u 4 = sin2 2θ


c4
 
i.e., 2 1 − 4 = (2 sin θ cos θ)2 = 4 sin2 cos2 θ
r
2 r − c4
4
= 4r4 sin2 cos2 θ


i.e., r4 − c4 = 2r4 sin2 θ cos2 θ


put x = r cos θ; y = r sin θ
2 2
2
− c4 = 2 (xy)2 = 2x2 y 2

i.e., x +y
i.e., x4 + y 4 + 2x2 y 2 − c4 = 2x2 y 2
x4 + y 4 = c4

This is the required equation of orbit.


Example 9 A particle subject to an acceleration
√ µ [2 (a2 + b2 ) u5 − 3a2 b2 u7 ] is
µ
projected at a distance a with velocity in a direction at right angles to the
a
initial distance. Show that the equation of the path is r2 = a2 cos2 θ + b2 sin2 θ.
Solution:
Given that F = µ [2 (a2 + b2 ) u5 − 3a2 b2 u7 ]
d2 u F µ [2 (a2 + b2 ) u5 − 3a2 b2 u7 ]
∴ u+ = =
dθ2 h2 u2 h2 u2
µ 
= 2 2 a2 + b2 u3 − 3a2 b2 u5
 
h
du
Multiplying both sides by 2 , we get

du du d2 u 2µ   du
2 a2 + b2 u3 − 3a2 b2 u5

2u +2 2
= 2
dθ dθ dθ h dθ
Integrating on both sides with respect to θ, we get
 2 Z
2 du 2µ 
2 a2 + b2 u3 − 3a2 b2 u5 du
 
u + = 2
dθ h
 4 6
 
2µ 2 2 u 2 2u
= 2 a +b − 3a b +A
h2 4 6
µ 
= 2 a2 + b2 u4 − a2 b2 u6 + A
 
(1.7)
h
16


µ √
Initially h = pv = po vo = a = µ
a
⇒ h2 = µ
µ
i.e., = 1
h2
1 1 du
Also, u = = and =0
r a dθ
1 (a2 + b2 ) a2 b2
∴ + 0 = − 6 +A
a2 a4 a
1 (a + b ) b2
2 2
i.e., = − 4 +A
a2 a4 a
1 a2 + b 2 − b 2
i.e., = +A ⇒ A=0
a2 a4
 2
2 du µ 
= 2 a2 + b2 u4 − a2 b2 u6
 
∴ u +
dθ h
 2
du  2
a + b2 u4 − a2 b2 u6 − u2 = u2 a2 + b2 u2 − a2 b2 u4 − 1
    
i.e., =

= u2 1 − b2 u2 a2 u2 − 1
  
i.e.,
du p
∴ = u [(1 − b2 u2 ) (a2 u2 − 1)]

du
i.e., = dθ
u [(1 − b2 u2 ) (a2 u2 − 1)]
1 1
put u = ; ∴ du = − 2 dr
r r
−1
dr
r2
s  = dθ
a2 b2

1
−1 1− 2
r r2 r
−rdr
i.e., p = dθ
(a2 − r2 ) (r2 − b2 )
put r2 = a2 cos2 φ + b2 sin2 φ;
2rdr = −2a2 cos φ sin φ + 2b2 sin φ cos φ
= b2 − a2 2 sin φ cos φdφ


i.e., − rdr = a2 − b2 sin φ cos φdφ



17

a2 − r 2 = a2 − a2 cos2 φ − b2 sin2 φ
a2 1 − cos2 φ − b2 sin2 φ

=
= a2 sin2 φ − b2 sin2 φ
a2 − b2 sin2 φ

=
2 2
a2 − b2 cos2 φ

similarly, r − b =
(a2 − b2 ) sin φ cos φdφ
∴ = dθ
(a2 − b2 ) sin φ cos φ
i.e., dφ = dθ
Integrating both sides , we get
Z Z
dφ = dθ
i.e, φ = θ + B
∴ r2 = a2 cos2 (θ + B) + b2 sin2 (θ + B)
If θ is measured from an apse, r = a and θ = 0, we get
a2 = a2 cos2 B + b2 sin2 B
i.e., a2 1 − cos2 B b2 sin2 B

=
i.e., a2 sin2 B = b2 sin2 B
a2 − b2 sin2 B

i.e., = 0
i.e., sin2 B = 0 ∵ a2 − b2 6= 0
i.e, B = 0
∴ r2 = a2 cos2 θ + b2 sin2 θ

Example 10 A√particle acted on by a central attractive force µu3 is projected


µ π
with a velocity at an angle with its initial distance a from the centre of
a 4
force. Show that the path is the equiangular spiral r = ae−θ .
Solution:
Given that F = µu3
d2 u F µu3
∴ u+ = =
dθ2 h2 u2 h2 u2
µu
=
h2
du
Multiplying both sides by 2 , we get

du du d2 u 2µu du
2u +2 =
dθ dθ dθ2 h2 dθ
18

Integrating on both sides with respect to θ, we get


 2 Z
2 du 2µ
u + = udu
dθ h2
2µ u2
= +A
h2 2
µ
= 2 u2 + A (1.8)
h
π a
Initially p = r sin φ ⇒ po = ro sin φo = a sin = √
4 2

a µ µ
h = pv = po vo = √ · ⇒ h2 =
2 a 2
µ
i.e., = 2
h2
 2
1 2 du
2
= u + = 2u2 + A
p dθ
2 2
= 2 +A ⇒ A=0
a a
 2
du
∴ u2 + = 2u2

 2
du
= u2

du
= u

du
i.e., = dθ
u
Integrating both sides, we get
log u = θ + log B
i.e., log u − log B + θ
u
i.e., log = θ
B
u
i.e., = eθ
B
i.e, u = Beθ
If θ is measured from an apse, r = a and θ = 0, we get
1
=B
a
1 θ
i.e., u = e
a
1 1 θ
i.e., = e ⇒ r = ae−θ
r a
19

2 Exercises
1. A particle describes a circular orbit under an attractive central force
directed towards a point on the circle. Show that the force varies as the
inverse fifth power of the distance.
[Hint: The polar equation of the circle is r = 2a cos θ.]
CHAPTER 6

MOMENT OF INERTIA
1 Definitions
Definition 1.1 (Moment of Inertia) If m is the mass of a particle and r
is its perpendicular distance from a given line, the quantity mr2 is called the
moment of inertia of the particle about the line.

Figure 1

If a series of particles of masses m1 , m2 , · · · etc., are arranged at points


whose perpendicular distances from a given straight line are r1 , r2 , · · · respec-
tively, then the quantity m1 r12 + m2 r22 + · · · is defined to be the moment of
inertia of the system of particles about the line and it is usually written as
mi ri2 .
P
If we write moment of inertia of a body of mass M about an axis as M k 2 ,
then k is called the radius of gryation of the body about the line.
Bookwork 1.2 (Parallel Axes Theorem:)
If I is the moment of inertia of a body about any axis and IG is its moment
of inertia about a parallel axis through G, the C.G. of body then I = IG + M h2 ,
where M is the mass of the body and h is the distance between the two parallel
axes.
Let us consider the case of a plane lamina. Let I denote the M.I. of a plane
lamina about the line Ox. Draw Oy perpendicular to Ox in the same plane.
Let G be the C.G. of the laminat with Coordinate (x, y) referred to Ox, Oy.
Consider an elementary mass m at the point P whose coordinates are (x, y)
referred to Ox, Oy. Let P be (X, Y ) referred to GX and GY a set of axes
parallel to Ox and Oy.
2

Figure 2

x = ON = OK + KN = OK + GM = x + X
y = N P = N M + M P = KG + M P = y + Y
I = M.I. of the lamina about Ox
X X X
= m.P N 2 = m.y 2 = m (y + Y )2
X
m. y 2 + 2y.Y + Y 2

=
X X X
= y2 m + 2y mY + mY 2 (1.1)

P
mY
Now P will the y-coordinate of the C.G. of the lamina with respect
m
to axes GX and GY . But it must be equal to zero, as G P itself is the origin in
the system. Hence the second term in (1.1) is zero. Also m = M.
X X
mY 2 = m.P M 2 = M.I. about GX = IG
y = GK = distance between the parallel axes Ox and Gx = h

Therefore, the equation (1.1) gives I = M h2 + IG

Bookwork 1.3 (Perpendicular Axes Theorem:) If Ix and Iy denote the


moments of inertia of a plane lamina about two rectangular axes Ox and Oy
in the plane, then Ix + Iy will be its moment of inertia about an axis through
O perpendicular to its plane.

Let Ox and Oy be two rectangular axes in the plane lamina and Oz be the
axis through O (not shown in the figure) perpendicular to the plane xOy.
Consider an elementary mass m at P .
3

Figure 3

Draw P M pernedicular to Ox.


X
Ix = M.I. of the lamina about Ox = m.P M 2
X
Iy = M.I. of the lamina about Oy = m.OM 2
X
Iz = M.I. of the lamina about Oy = m.OP 2
X  X X
= m. P M 2 + OM 2 = m.P M 2 + m.OM 2 = Ix + Iy

2 Moments of Inertia in some particular cases:


1. A thin uniform rod
Let AB be a thin uniform rod of length 2a. O is the midpoint and OY is
perpendicular to AB. Let us find the M.I. of the rod about OY .

Figure 4

Consider an elementary section P Q of the rod, of length 4x at a distance x


from O.
4

Let ρ be the mass per unit length of the rod.


Mass of the section P Q = ρ · 4x.
M.I. of elementary section P Q about OY = ρ · 4x · x2 .
x=a
X
Hence M.I. of rod AB about OY = ρ · 4x · x2
x=−a
Z x=a a
x3

2
= ρx dx = ρ
x=−a 3 −a
 3  3 
a a 2a3
= ρ − − =ρ·
3 3 3
3
 
M 2a M
= · ∵ρ=
2a 3 2a
2
Ma
=
3

Note 2.1 M.I. of the rod AB about the line passing through A perpendicular
to the rod AB=M.I. of rod about OY +M.(OA)2 (By using parallel axes
theorem)
M a2 2 4M a2
= + Ma =
3 3
2. Uniform Circular Ring
(a). About an axis through the centre perpendicular to its plane.

Figure 5

Let a be the radius of the ring. Then each particle of the ring is at the same
distance a from the axis perpendicular to the plane through the centre.
5

Hence M.I. of the elementary


X mass = mass × a2 .
mass × a2 = M a2 where M is the mass of

Thus M.I. of the ring =
the ring.
(b) About a diameter.

Figure 6

Let I be the M.I of ring about a diameter.


By symmetry, I is also the M.I. of ring about any other diameter.
Take any two perpendicular diameters. Then by perpendicular axes theo-
rem,

I + I = M.I. of the ring about an axis through the centre perpendicular to the plane
= M a2
2I = M a2
M a2
i.e., I =
2
(c) About a tangent line.

Figure 7
6

By parallel axes theorem,

M.I.of the circular ring about a tangent line = M.I.about a parallel diameter + M a2
M a2 3M a2
= + M a2 =
2 2
3. Uniform Circular Disc

Figure 8

Let O be the centre , a be the radius of circular disc and ρ be the mass per
unit area.
Divide the circular disc into a concentric rings of radii x and x + 4x,
breadth is 4x.
Thus the area contained between two concentric rings of radii x and x+4x
is 2πx · 4x.
Mass of the elementary area is 2πρx · 4x.
x2
M.I. of the circular ring about a diameter = 2πρx · 4x · = πρx3 · 4x.
2
Hence, M.I. of circular disc about a diameter x = a is
x=a
X
= πρx3 4x
x=0
Z x=a
= πρ x3 dx
x=0a
x4 a4
= πρ = πρ ·
4 0 4
4
M a
∵ M = πa2 ρ

= π· 2 ·
πa 4
M a2
=
4
7

M a2
By symmetry, M.I. of the disc about a perpendicular diameters is also .
4
Hence by perpendicular axes theorem, M.I. of a circular disc about an axis
M a2 M a2 M a2
through the centre perpendicular to the disc = + = .
4 4 2
Note 2.2 M.I. of circular disc about a tangent line = M.I. of circular disc
about a parallel diameter+M a2 .
M a2 5M a2
= + M a2 = .
4 4
4. Uniform elliptic lamina:
(a). About its Major axes:

Figure 9

Let P (x, y) be any point on the ellipse.


Divide the area into a elementary rectangular strips perpendicular to major
axis.
Area of elementary section of length 2y and breadth 4x is = 2y4x.
If ρ is the mass per unit area, then mass of the elementary section is 2yρ4x.
M.I. of elementary section about a line through the centre is
y2 y3
2yρ4x · = 2ρ 4x.
3 3
x=a
y3 4ρ x=a 3
X Z
M.I. of ellipse about a major axis is = 2 2ρ 4x = y dx
x=0
3 3 x=0

Put x = a cos θ and y = b sin θ


4ρ 0 3 3
Z
M.I. = b sin θ · (−a sin θdθ)
3 π/2
4ρab3 π/2 4
Z
= sin θdθ
3 0
4ρab3 3 1 π πρab3
= · · · =
3 4 2 2 4
8

πab3 M
= · (∵ M = πρab)
4 πab
2
Mb
=
4
(b). About its Minor axes:

Figure 10

Let P (x, y) be any point on the ellipse.


Divide the area into a elementary rectangular strips perpendicular to minor
axis.
Area of elementary section of length 2x and breadth 4y is = 2x4y.
If ρ is the mass per unit area, then mass of the elementary section is 2xρ4y.
M.I. of elementary section about a line through the centre is
x2 x3
2xρ4y · = 2ρ 4y.
3 3
y=b
x3 4ρ y=b 3
X Z
M.I. of ellipse about a minor axis is = 2 2ρ 4y = x dy
y=0
3 3 y=0

Put x = a cos θ and y = b sin θ


4ρ π/2 3
Z
M.I. = a cos3 θ · (b cos θdθ)
3 0
4ρba3 π/2
Z
= cos4 θdθ
3 0
4ρba3 3 1 π πρba3
= · · · =
3 4 2 2 4

πba3 M
= · (∵ M = πρab)
4 πab
2
Ma
=
4
9

Note 2.3 M.I. of elliptic lamina about


 2 a line
 through the centre perpendicular
M a2 M b2 a + b2
to the plane = + =M .
4 4 4

5. A Solid Sphere about its diameter:

Figure 11

Let a be the radius of the sphere.


Divide the solid sphere into a circular disc of radius y.
Consider the elementary section at a distance x from the centre of the
sphere.
Volume of typical circular disc of radius y and thickness 4x is πy 2 4x.
Let ρ be the mass per unit volume.
Thus, Elementary mass of the typical circular disc is ρπy 2 4x.
M.I. of a circular disc about a line passing through the centre is

y2
= ρπy 2 4x ·
2

Hence, M.I. of a solid sphere about its diameter is


x=a
X y2
= ρπy 2 4x ·
x=−a
2
Z x=a
y2
= ρπy 2 · dx
x=−a 2
Z a
= πρ y 4 dx
Z0 a
2
a2 − x 2 on the circle x2 + y 2 = a2

= πρ dx
0
10

Put x = a cos θ; dx = −a sin θ, we get


Z 0
2
M.I. = πρ a2 − a2 cos2 θ (−a sin θ) dθ
π/2
Z π/2
= πρ a5 sin5 θdθ
0
4 2 πρ
= πρa5 · · = · 8a5
5 3 15  
π 5 M 4 3
= · 8a ∵ M = ρπa
15 4 3 3
πa
3
π 3M 2M a2
= · 8a5 =
15 4πa3 5

Note 2.4
M.I. of the sphere about a tangent line
= M.I. about a parallel diameter +M a2
2M a2 7M a2
= + M a2 =
5 5
6. A Hollow Sphere about its diameter:

Figure 12

Divide the hollow sphere into a thin circular rings by planes perpendicular to
axis.
Consider a typical ring of radius y and arcual width 4s, at a distance x
from O.
Elementary surface area of typical ring = 2πy · 4s.
Let ρ be the mass per unit area of the surface.
11

Elementary Mass = ρ · 2πy · 4s.


M.I. of circular ring about OX = ρ × 2πy4s · y 2 = 2πρy 3 4s.

M.I. of hollow sphere about OX


x=a
X
= 2π · ρy 3 4s
x=−a
Z x=a
= 2π · ρy 3 ds
Zx=−a
a
= 4πρy 3 ds
0
dy
But x2 + y 2 = a2 ⇒ 2x + 2y =0
dx
dy x
i.e., = −
dx y
s 2 s
x2

ds dy
= 1+ = 1+
dx dx y2
s s
x2 + y 2 a2 a
= = =
y2 y 2 y
a
∴ ds = dx
y
Z a
a
Hence, M.I. of hollow sphere about OX = 4πρy 3 dx
y
Z0 a
= 4aπρy 2 dx
Z0 a
4πρa a2 − x2 dx

=
0
a
x3

2
= 4aπρ a x −
3 0
3
 
3 a
= 4aπρ a −
3
3
2a 8a4 πρ
= 4aπρ =
3 3
8a4 π M 2

= · ∵ M = 4πa ρ
3 4πa2
2
2M a
=
3
12

7. A Solid right circular cone about its axis:

Figure 13

Let h be the height of the cone, r be the base radius, V be the vertex of
the cone.
Divide the solid right circular cone into a circular discs.
Consider an elementary section, P P 0 be a circular section at a distant x
from the vertex, y be the radius of the circular section and thickness 4x.
Volume of elementary section = πy 2 4x.
Let ρ be the mass per unit volume.
Mass of the elementary section = ρπy 2 4x.
M.I. of elementary section about a line passing through the centre perpen-
y2
dicular to the plane = ρπ · y 2 · 4x · .
2
Hence, M.I. of solid cone about V M is
x=h
X y2
= ρπ · y 2 · 4x ·
x=0
2
Z h
ρπ
= y 4 dx
2 0

Triangles V LP and V M B are similar.


LP VL y x rx
Hence = i.e, = ⇒ y=
MB VM r h h
Z h 4 4
ρπ r x
M.I. of solid cone = dx
2 0 h4
ρπr4 h5
= ·
2h4 5
13

πr4 h
 
M 1 2
= · ∵ M = πr hρ
10 1 2 3
πr h
3
πr4 h 3M 3M r2
= · 2 =
10 πr h 10

8. A hollow right circular cone about its axis:

Figure 14

Let h be the height, r be the base radius and V be the vertex of the hollow
right circular cone.
Divide the hollow cone into a circular rings.
Consider the typical section at a distance x from the vertex and arcual
width = 4s.
Surface Area of the elementary section = 2πy · 4s.
Let ρ be the mass per surface density.
M.I. of elementary section about the line through the centre perpendicular
to the plane = ρ · 2πy4s · y 2 = 2πρy 3 4s.
Hence, the M.I. of hollow cone

x=h
X Z h
3
= 2πρy 4s = 2πρ y 3 ds
x=0 0

y x rx
By Similar triangles = ⇒ y=
r √r h h
ds r2 r2 + h2 l
= 1+ 2 = = , where l is the slant height of the cone.
dx h h h
14

Hence, the required M.I. of required cone is


Z h 3 3
r3 l h 3
Z
r x l
= 2πρ · dx = 2πρ · 4 x dx
0 h3 h h 0
 h
r 3 l x4
= 2πρ · 4
h 4 0
πρ 3
= r l
2
π 3 M
= r l· (∵ M = πrlρ)
2 πrl
M r2
=
2

Example 1 Show that the M.I. of a triangular lamina of mass M about a side
M h2
is where h is the altitude from the opposite vertex.
6
Solution:

Figure 15

Let ABC be a triangular lamina.


Divide the lamina into strips parallel to BC. Let P Q be the elementary
strip at a distance x from A and its widths is 4x.
Thus, area of the strip = P Q · 4x.
PQ AL x ax
By similar triangles, = = ⇒ PQ = , where BC = a.
BC AD h h
Let ρ be the areal denisty.
ρax
Mass of the strip P Q= 4x.
h
M.I. of strip P Q about BC = M.I. of P Q about a line through the midpoint
of P Q parallel to BC+(mass of P Q)(LD)2
15

=M.I. of P Q about itself +(mass of P Q)(LD)2


ax
= 0+ρ 4x (h − x)2
h
Hence the M.I. of 4ABC about its base BC
x=h
X ρax
= (h − x)2 4x
x=0
h
Z h
ρa
x h2 − 2hx + x2 dx

=
h 0
 2 2 h
ρa h x x3 x4
= − 2h +
h 2 3 4 0
 4 4 4

ρa h 2h h
= − +
h 2 3 4
4 3
ρa h ρah
= · =
h 12 12 
ah3 M

1
= · ∵ M = ρah
12 1 2
ah
2
ah3 2M M h2
= · =
12 ah 6

You might also like